Subscribe to ForumIAS

UPSC Previous Year Questions: CSP 2021

Meanwhile, to practice questions not asked previously, utilize this as well to the max!

https://forumias.com/post/detail/10000-Questions-for-Prelims-2020-Prelims-2020-Marathon-1576210932

30.6k views

According to the Constitution of India, it is the duty of the President of India to cause to be laid before the Parliament which of the following? 

1. The Recommendations of the Union Finance Commission

2. The Report of the Public Accounts Committee

3. The Report of the Comptroller and Auditor General

4. The Report of the National Commission for Scheduled Castes

Select the correct answer using the codes given below

a. 1 only

b. 2 and 4 only

c. 1, 3 and 4 only

d. 1, 2, 3 and 4

33.3k views

The members of Constituent Assembly were

a. Elected by the provincial assemblies

b. Elected directly by the people

c. Nominated by the government

d. Only representatives of princely states

 

33.4k views

Which one of the following statements is correct?

a. In photosynthesis by Green plants, CO2 is used for making carbohydrate. 

b. In photosynthesis by Green plants, plants use oxygen for making carbohydrate. 

c. Carbohydrate generated by plants comes from soil.

d. Carbohydrate generated by plants comes from urea.


(CAPF 2018)

30.5k views

Barak Valley is famous for the cultivation of

a. Tea

b. Sugarcane

c. Jute

d. Cotton

(CSE)

30.5k views

@root @Neyawn

30.5k views

Why did Buddhism started declining in India  in early medieval times ?

1. Buddha was by that time considered as one of the incarnations of Vishnu and thus became a part of Vaishnavism.

2. Invading tribes from Central Asia till the time of last Gupta king adopted Hinduism and persecuted Buddhists.

3. Kings of Gupta dynasty were strongly opposed to Buddhism.

Which of the above given statements are correct?

A. 1 only 

B. 1 and 3 only 

C. 2 and 3 only 

D. 1, 2 and 3

30.6k views

Any takers ?

30.5k views

Logic for Barak valley tea or jute? m feeling sceptical.

Even I am unable to understand the question in totality. Some websites claim Jute is not dominant there as jute is more of a delta crop. Therefore by elimination, sugarcane/teais the best option

30.5k views

FCsaid


@upsc2020 Good initiative...pls number the questions

1-d, 2-a, 3-a, 4-b, 5-a

Sure. Thanks. 

30.5k views

6. If the Prime Minister of India belonged to the Upper House of Parliament

a) he will not be able to vote in his favour in the event of a no-confidence motion.

b) he will not be able to speak on the budget in the Lower House

C) he can make statements only in the Upper House

D) he has to become a member of the Lower House within six months after being sworn in as the Prime Minister. 

(CSE 97)

30.5k views

7.  King Cobra is the only snake that makes its own nest. Why does it make its nest?


A. It is a snake-eater and the nest helps attract other snakes


B. It is a viviparous snake and needs a nest to give birth to its offspring


C. It is an oviparous snake and lays its eggs in the nest and guards the nest until they are hatched


D. It is a large, cold blooded animal and needs a nest to hibernate in the cold season.




CSE 2010

30.5k views

@FC 1-c. Rest all correct.


Any suggestions as to how you would want me to upload solutions? After every five questions I will upload the answers. 

30.5k views

1 c

2 a

3b

4c

5a

1-c, 2-a, 3-a, 4-a, 5-a

30.5k views

FCsaid

@upsc2020 okay thanks...
I think before you upload the next set of questions, upload the answers for previous set ...as you said.


Sure !

5.1k views

6a 7c

Correct.


6 A

7 C

5.1k views

8. Among the following, who are the Agaria community?


A. A traditional toddy tappers community of Andhra Pradesh

B. A traditional fishing community of Maharashtra

C. A traditional silk-weaving community of Karnataka

D. A traditional salt pan workers community of Gujarat


CSE 2009

5.1k views

9. With reference to the evolution of living organisms, which one of the following sequences is correct?

A. Octopus-Dolphin-Shark     

B. Pangolin- Tortoise-Hawk

C. Salamander-Python-Kangaroo

D. Frog-Crab-Prawn

5.1k views

10. After Quit India Movement, C. Rajagopalachari issued a pamphlet entitled “The Way Out”. Which one of the following was a proposal in this pamphlet?

A. The establishment of a "War Advisory Council" composed of representatives of British India and the Indian States.

B. Reconstitution of the Central Executive Council in such a way that all its members, except the Governor General and the Commander-in- Chief should be Indian leaders.

C. Fresh elections to the Central and Provincial Legislatures to be held at the end of 1945 and the Constitution making body to be convened as soon as possible.

D. A solution for the constitutional deadlock.


(CSE)

5.1k views

11. Though coffee and tea both are cultivated on hill slopes, there is some difference between them regarding their cultivation.

In this context, consider the following statements:

1. Coffee plant requires a hot and humid climate of tropical areas whereas tea can be cultivated in both tropical and subtropical areas.

2. Coffee is propagated by seeds but tea is propagated by stem cuttings only.

Which of the statements given above is/ are correct?

A. 1 only

B. 2 only

C. Both 1 and 2

D. Neither 1 nor 2


(cse)

5.1k views

12. Following are the characteristics of an area in India:

1. Hot and humid climate

2. Annual rainfall 200 cm

3. Hill slopes up to altitude of 1100 metres

4. Annual range of temperature 15°C to 30°C

Which one among the following crops are you most likely to find in the area described above?

A. Mustard

B. Cotton

C. Pepper

D. Virginia tobacco


(cse)

5.1k views

13. If there were no Himalayan ranges, what would have been the most likely geographical impact on India?

1. Much of the country would experience the cold waves from Siberia.

2. Indo-Gangetic plain would be devoid of such extensive alluvial soils.

3. The pattern of monsoon would be different from what it is at present.

Which of the statements given above is/are correct?

A. 1 only

B. 1 and 3 only

C. 2 and 3 only

D. 1, 2 and 3

(cse)

5.1k views

14. There are only two known examples of cave paintings of the Gupta period in ancient India. One of these is paintings of Ajanta caves. Where is the other surviving example of Gupta paintings?

A. Bagh caves

B. Ellora caves

C. Lomas Rishi cave

D. Nasik caves


(CSE)

5.1k views

15. Sustainable development is described as the development that meets the needs of the present without compromising the ability of future generations to meet their own needs. In this perspective, inherently the concept 'of sustainable development is intertwined with which of the following concepts?

A. Social justice & empowerment

B. Inclusive Growth

C. Globalization

D. Carrying capacity


(CSE)

5.1k views

8d 9  10 d  11c 12c 13d  14 a 15 b


pls elaborate how to approach 9.

8 - D

9 - C

10 - D

11- A (Budding, grafting also used for Coffee)

12 - C

13 - D

14 - A

15 - D

5.1k views

8d 9  10 d  11c 12c 13d  14 a 15 b


pls elaborate how to approach 9.

Approach for Q.9 should be this diagram. Salamander being an amphibian, comes before a python, which is a reptile. evolution | Theory, Examples, & Facts | Britannica

6k views

16. Which of the following factors affects individual’s demand for a commodity?

  1. Price of the commodity
  2. Income of the consumer
  3. Prices of related goods

Select the answer using the code given below:

a. 1 and 2 only

b. 2 and 3 only

c. 1, 2 and 3

d. 1 only


(CAPF)

5.1k views

17. The rate at which the consumer is willing to substitute one good for another without changing the level of satisfaction is known as:

a. Marginal rate of substitution

b. Marginal rate of technical substitution

c. Diminishing marginal utility

d. Equi-marginal utility


(CAPF)

4.2k views

18. Consider the following statements :

A Constitutional Government is one which

1. Places effective restrictions on individual liberty in the interest of State Authority

2. Places effective restrictions on the Authority of the State in the interest of individual liberty

Which of the statements given above is / are correct?

a. 1 only

b. 2 only

c. Both 1 and 2

d. Neither 1 nor 2


(CSE)

4.2k views

19. Public Interest Litigation falls within the jurisdiction of the High Court provided certain conditions are fulfilled. Which one among the following conditions is not accepted by the courts?

a. Public is interested in vindication

b. Enforcement of public Duty

c. Courts can examine previous records of public servant.

d. Personal injury or loss is an essential element.


(CAPF)

4.2k views

20. Which one among the following is the nearest from the ‘Zero Mile Centre’ of India?

a. Taj Mahal (Agra)

b. Charminar (Hyderabad)

c. Gateway Of India (Mumbai)

d. Victoria Memorial (Kolkata)


(CAPF)

4.2k views

21. Bleaching powder is added in water used for drinking purpose. The role of bleaching powder in this case is

a. of disinfectant

b. of bleaching agent

c. to remove impurity

d. to decrease the pH


(CAPF)

4.2k views

22. Which of the following practices can help in water conservation in agriculture? ‎

‎1. Reduced or zero tillage of the land ‎

‎2. Applying gypsum before irrigating the field ‎

‎3. Allowing crop residue to remain in the field ‎

Select the correct answer using the code given below : ‎

a. 1 and 2 only‎

b. 3 only ‎

c. 1 and 3 only ‎

d. 1, 2 and 3


(CSE)

4.2k views

If possible post 2 pvs yr question csat too daily ..this platform increases accountability.

Sure. I will.

4.2k views

16. C 

17. A

18. B

19. D

20. B

21. A

22. C

4.2k views

16 c 

17 a

18 b 

19 dint understand question in totality..or I  need more patience don't know.

20 no idea at all..tell me how to approach logically.

21a

22a

For question 19: 

It essentially wants us to mark out the statement that isincorrectwith respect to a PIL. 

When a PIL is filed, the basic premise is that the court must be careful to see that the petitioner is not doing it for personal gain, private profit or political or other considerations.


4.2k views

16 c 

17 a

18 b 

19 dint understand question in totality..or I  need more patience don't know.

20 no idea at all..tell me how to approach logically.

21a

22a

For Q:20

Nagpur lies precisely at the center of the country with the Zero Mile Marker which indicates the geographical center of India. 

So for this question, you have to know two facts: 

1. Nagpur is called zero centre

2. You have seen the location of major cities on Atlas.


7k views

23. The famous terra-cotta images of rivers Ganga and Yamuna from the Gupta period have been found at

A. Ahichchhatra

B.Kaushambi

C. Rajghat

D. Bhitargaon

4.2k views
23 a

Right. 

23. A

4.4k views

24. In India, if a species of tortoise is declared protected under Schedule I of the Wildlife (Protection) Act, 1972, what does it imply ?

(a) It enjoys the same level of protection as the tiger.

(b) It no longer exists in the wild, a few individuals are under captive protection; and now it is impossible to prevent its extinction.

(c) It is endemic to a particular region of India.

(d) Both (b) and (c) stated above are correct in this contex

4.4k views

25. Due to some reasons, if there is a huge fall in the population of species of butterflies, what could be its likely consequence/consequences?

1. Pollination of some plants could be adversely affected.

2. There could be a drastic increase in the fungal infections of some cultivated plants.

3. It could lead to a fall in the population of some species of wasps, spiders and birds.

Select the correct answer using the code given below:

(a) 1 only

(b) 2 and 3 only

(c) 1 and 3 only

(d) 1, 2 and 3

4.4k views

26. The release of which one of the following into ponds and wells helps in controlling the mosquitoes ?

(a) Crab                     

(b) Dogfish

(c) Gambusia fish    

 (d) Snail

4.4k views

27. Mixture of which one of the following pairs of gases is the cause of occurrence of most of the explosions in mines?

(a) Hydrogen and oxygen

(b)Oxygen and acetylene

(c)  Methane and air

(d) Carbon dioxide and methane

4.4k views

Nikitasaid

Gambusia fish.

Known as guppies. I have a small bowl in my study room. They are small, cost 20 rupees a pair and multiply like anything. They dont  need the pump and all unlike the gold fish which dies in the bowl.


Is this a upsc previous year question?




26. The release of which one of the following into ponds and wells helps in controlling the mosquitoes ?

(a) Crab                     

(b) Dogfish

(c) Gambusia fish    

 (d) Snail


Yes, all questions posted on this thread are previous year questions.

4.4k views

pk2901said

@upsc2020 Should be (c) - Methane and Air for Q.27 bcs Carbon Dioxide suppresses fire.


Correct.

Methane explosions occur in mines when a buildup of methane gas, a byproduct of coal, comes into contact with a heat source, and there is not enough air to dilute the gas to levels below its explosion point.

4.1k views

32. Several nationalist leaders in India wrote commentaries on the Bhagavad Gita to argue the case for an ethical foundation to Indian nationalism, Who among the following is an exception to it?

a. Sri Aurobindo

b. Mahatma Gandhi

c. Bal Gangadhar Tilak

d. Ram Manohar Lohia

4.1k views

33. How does participatory-budgeting seek to make the functioning of local governance institutions more transparent and accountable?

  1. By allowing citizens to deliberate and negotiate over the distribution of public resources.
  2. By allowing citizens to play a direct role in deciding how and where resources should be spent.
  3. By allowing historically excluded citizens with access to important decision-making venues

Select the correct answer using the codes given below.

a. 1 and 2

b. 2 and 3

c. Only 3

d. 1, 2 and 3

4.1k views

34. Various landforms observed on the surface of the earth are due to the:

  1. differences in the type and structure of the crustal material
  2. differences in the land forming processes
  3. differential rates of the processes at different places on the Earth’s surface

Select the correct answer using the code given below:

a. 1 only

b. 1 and 2 only

c. 2 and 3 only

d. 1, 2 and 3

4.1k views

35. From which among the following pairs of species, a small quantity of fine quality wool is obtained in India?

a. Pashmina goats and Angora rabbits

b. Pashmina rabbits and Angora goats

c. Pashmina rabbits and Angora sheep

d. Pashmina goats and Angora sheep

4.1k views

36. Trees of tropical rainforest have buttress roots because :

a. they help to provide aeration to soils

b. the organisms found in the buttresses have a symbiotic relationship

c. the trees belong to gramineae family

d. the buttresses have to bear the mechanical load of hardwoods

4.1k views

32. Several nationalist leaders in India wrote commentaries on the Bhagavad Gita to argue the case for an ethical foundation to Indian nationalism, Who among the following is an exception to it?

a. Sri Aurobindo

b. Mahatma Gandhi

c. Bal Gangadhar Tilak

d. Ram Manohar Lohia

D?

32 D


Correct@Patrick_jane 

4.1k views

37. Who among the following were well known as champions of women’s education in colonial India ?

  1. Sister Subbalaksmi
  2. Begum Rokeya Sakhawat Hossain
  3. Keshub Chandra Sen
  4. Ananda Coomaraswamy

Select the correct answer using the code given below:

a. 1, 2, 3 and 4

b. 1, 2 and 3 only

c. 3 and 4 only

d. 1 and 2 only

4.1k views

Answers pending from users for 33-36 !!

4.1k views

33- d, 34- d, 35- a

Correct. 

33 D 

34 D 

35 A 

36 D

4.4k views

Gochisaid

37. Who among the following were well known as champions of women’s education in colonial India ?

  1. Sister Subbalaksmi
  2. Begum Rokeya Sakhawat Hossain
  3. Keshub Chandra Sen
  4. Ananda Coomaraswamy

Select the correct answer using the code given below:

a. 1, 2, 3 and 4

b. 1, 2 and 3 only

c. 3 and 4 only

d. 1 and 2 only

B?

B is correct. 


37 B

4.4k views

38)

CDS 2017

A

4.4k views

39)

CDS 2017

D

4.4k views

40)

CDS 2017

A? could be D - as points are interrelated. Not sure. 

4.4k views

41. Consider the following statements pertaining to the Satavahanas:

1. Satavahana rulers were identified through metronymics

2. Satavahana, succession to the throne was generally patrilineal

Which of the statements given above is/are correct?

(a) 1 only

(b) 2 only

(c) Both 1 and 2

(d) Neither 1 nor 2

4.4k views

42. Who among the following were jailed in the Kanpur Bolshevik conspiracy case in 1924?


(a) Muzaffar Ahmad, S. A. Dange, Shaukat Usmani, Nalini Gupta


(b) Muhammad Ali and Shaukat Usmani


(c) S. A. Dange and S. V. Ghate


(d) Muzaffar Ahmad and S. S. Mirajkar

4.4k views

43. The Sufi work Kashf-ul-Mahjub was written by


(a) Abu’l Hasan al Hujwiri


(b) Moinuddin Chishti


(c) Shaikh Nizamuddin Auliya


(d) Amir Khusrau

4.4k views

44. Which one of the following does NOT fall in the areas of tropical deciduous forest biomes?


(a) The Neotropics mainly West Indies


(b) Indo-Malaysian zone mainly in south and south-east Asia except equatorial evergreen rainforest areas


(c) Eastern Africa and northern, Australia


(d) North America mainly South-west America

4.5k views

45. Doldrums are


1. equatorial calms


2. calm and light winds


3. roaring forties


4. variable both in position and in extent


Select the correct answer using the code given below:


(a) 1, 2 and 4


(b) 2 and 3 only


(c) 1 and 3


(d) 2, 3 and 4

4.4k views

46. Which one of the following authorities ing authorities CANNOT organize ‘Lok Adalats’?


(a) Panchayat Committees


(b) Taluk Legal Services Committee


(c) District Authority


(d) High Court Legal Services Committee

4.4k views

41-c, 42-a, 43-a, 44-d, 45-a, 46-a ?

41 C

42 A

43 A

44 A

45 A

46 A

@singhpradyumna125831 

4.3k views

If a commodity is provided free to the public by the Government, then

(a) The opportunity cost is zero.

(b) The opportunity cost is ignored.

(c) The opportunity cost is transferred from the consumers of the product to the tax-paying public.

(d) The opportunity cost is transferred from the consumers of the product to the Government.

Though the answer given is C ....however question arise that the govt might use debt finance to provide free service, in that case taxpaying public doesn't share the burden ....so D is plausible?

One of the contentious questions. We assume/ and the fact is that the major source of revenue is taxes collected by public. What is provided free to us? Street Light, protection, public parks etc etc. But where does the money come to the government eventually to maintain these? Thus, I would go with C. 

4.3k views

41 a

43a chisti Nizamuddin not much major books ..khusrau read in detail so not connected via past knowledge..finally after chintan intuition hitting hujwiri.

42a

44b

45a

46d?how to eliminate??

For 46: Lok Adalat is an important topic. NLSA website gives out the important details and question can be solved from there by elimination. 


Levels and Composition of Lok Adalats:


At the State Authority Level -

The Member Secretary of the State Legal Services Authority organizing the Lok Adalat would constitute benches of the Lok Adalat, each bench comprising of a sitting or retired judge of the High Court or a sitting or retired judicial officer and any one or both of- a member from the legal profession; a social worker engaged in the upliftment of the weaker sections and interested in the implementation of legal services schemes or programmes.

At High Court Level -

The Secretary of the High Court Legal Services Committee would constitute benches of the Lok Adalat, each bench comprising of a sitting or retired judge of the High Court and any one or both of- a member from the legal profession; a social worker engaged in the upliftment of the weaker sections and interested in the implementation of legal services schemes or programmes.

At District Level -

The Secretary of the District Legal Services Authority organizing the Lok Adalat would constitute benches of the Lok Adalat, each bench comprising of a sitting or retired judicial officer and any one or both of either a member from the legal profession; and/or a social worker engaged in the upliftment of the weaker sections and interested in the implementation of legal services schemes or programmes or a person engaged in para-legal activities of the area, preferably a woman.

At Taluk Level-

The Secretary of the Taluk Legal Services Committee organizing the Lok Adalat would constitute benches of the Lok Adalat, each bench comprising of a sitting or retired judicial officer and any one or both of either a member from the legal profession; and/or a social worker engaged in the upliftment of the weaker sections and interested in the implementation of legal services schemes or programmes or a person engaged in para-legal activities of the area, preferably a woman.

National Lok Adalat

National Level Lok Adalats are held for at regular intervals where on a single day Lok Adalats are held throughout the country, in all the courts right from the Supreme Court till the Taluk Levels wherein cases are disposed off in huge numbers. From February 2015, National Lok Adalats are being held on a specific subject matter every month.

4.3k views

47. Why did the early nationalists oppose the Council Act of 1892?


(a) The number of members of the Imperial Legislative Council was increased


(b) The number of members of Provincial Councils was increased


(c) The Councils were given the right to discuss the annual budgets


(d) The Act did not give the Indians control over the public funds

4.3k views

48. Which of the following conditions is/are essential for wheat cultivation?


1. Optimum temperature during growing period is around 30°C


2. A frost-free period of about 100 days


3. Light clay or heavy loam soil


Select the correct answer using the code given below:


(a) 1, 2 and 3


(b) 1 and 2 only


(c) 2 and 3 only


(d) 1 only

4.3k views

49. Which of the following are the defining features of liberal democracy?


1. Constitutional government based on formal, usually legal, rules


2. Guarantees of civil liberties and individual rights


3. It invests political authority with potentially unlimited power


Select the correct answer using the code given below:


(a) 1 and 3 only


(b) 1 and 2 only


(c) 2 and 3 only


(d) 1, 2 and 3

4.3k views

50.  Which one of the following is an example of a price ceiling?


(a) Fares charged by Airlines in India


(b) Price printed on biscuit packets


(c) Minimum support price for cane growers


(d) Minimum wages fixed by state Governments

4.3k views

47 D

48 A

49 B

50 B


47 D

48 C

49 B

50 C- However, I would request some inputs here. 

4.3k views

51. The Reserve Bank of India regulates the commercial banks in matters of

  1. liquidity of assets
  2. branch expansion
  3. merger of banks
  4. winding-up of banks

Select the correct answer using the codes given below.

a. 1 and 4 only

b. 2, 3 and 4 only

c. 1, 2 and 3 only

d. 1, 2, 3 and 4

4.3k views

52. National income of a country for a given period is equal to the

a. total value of goods and services produced by the nationals

b. sum of total consumption and investment expenditure

c. sum of personal income of all individuals

d. money value of final goods and services produced

4.3k views

53. If the interest rate is decreased in an economy, it will

a. decrease the consumption expenditure in the economy

b. increase the tax collection of the Government

c. increase the investment expenditure in the economy

d. increase the total savings in the economy

4.2k views

47 b

48d

49b

50d analysis for price ceiling examples?

51d

52d

53c

47 D

48 C

49 B

50 C 

51 D

52 D

53 C


4.3k views

47 b

48d

49b

50d analysis for price ceiling examples?

51d

52d

53c

For Price Ceiling question, what I understand is:

Price Ceiling: a legal maximum on the price of a good or a service. For example, government set a price ceiling for Uber to give taxis back an advantage. By putting a ceiling on the price of Uber rides, govt attempted to lower the high price of Uber rides during peak times. However, that didn't happen. The taxi never came, and Uber would show it as shortage. 

Whenever a government sets a price ceiling, shortages occur - the demand is larger then what the seller is willing to provide.

Feel free to ask if the concept isn't clear. 


4.3k views

54. The concept of Anuvrata was advocated by

a. Mahayana Buddhism

b. Hinayana Buddhism

c. Jainism

d. Lokayata School

4.3k views

55. According to Mimansa system of philosophy, liberation is possible by means of

a. Gyana

b. Bhakti

c. Karma

d. Yoga

4.3k views

56. According to ancient Indian cosmogenic ideas, the sequential order of the cycle of four yugas is

a. Dvapara, Krita, Treta and Kali

b. Krita, Dvapara, Treta and Kali

c. Krita, Treta, Dvapara and Kali

d. Treta, Dvapara, Kali and Krita

4.1k views

C

For which question?

4.2k views

54 C

55 C

56 C

4.2k views

54c means little vows practiced by Jain's.For example - not killing animals.

55a

56b

54 C

55 C

56 C

4.1k views

57. Which one of the following was not proposed by the 73rd Constitutional Amendment of Panchayati Raj?


(a) Thirty percent seats in all elected rural local bodies will be reserved for women candidates at all level


(b) The States will constitute their Finance Commissions to allocate resources to Panchayati Raj institutions


(c) The Panchayati Raj functionaries will be disqualified to hold their offices if they have more than two children


(d) The elections will be held in six months time if Panchayati Raj bodies are superceded or dissolved by the State government

4.1k views

58. Which one of the following duties is not performed by Comptroller and Auditor general of India?


(a) To audit and report on all expenditure from the Consolidated Fund of India


(b) To audit and report on all expenditure from the Contingency Funds and Public Accounts


(c) To audit and report on all trading, manufacturing, profit and loss accounts


(d) To control the receipt and issue of public money, and to ensure that the public revenue is lodged in the exchequer

4.1k views

59. The Parliament can make any law for whole or any part of India for implementing international treaties

A. without the consent of any State

B. with the consent of all the States

C. with the consent of the State concerned

D. with the consent of the majority of States

4.1k views

60. As an alternative to the partition of India, Gandhiji suggested to Mountbatten that he :

A.postponed granting of independence

B. Iinvited Jinnah to form the government

C. Invited Nehru and Jinnah to form the government together

D. invite the army to take over for some time

4.1k views

57c

58d

59a

60b

All Correct. 

57c

58d

59a

60b

4.1k views

61. Westerlies in southern hemisphere are stronger and persistent than in northern hemisphere. Why?

  1. Southern hemisphere has less landmass as compared to northern hemisphere.
  2. Coriolis force is higher in southern hemisphere as compared to northern hemisphere

Which of-the statements given above is/are correct?

(a.) 1 only


(b.) 2 only


(c.) Both 1 and 2


(d.) Neither 1 nor 2

4.1k views

62. Normally, the temperature decreases with the increase in height from the Earth’s surface, because


  1. the atmosphere can be heated upwards only from the Earth’s surface
  2. there is more moisture in the upper atmosphere
  3. the air is less dense in the upper atmosphere

Select the correct answer using the codes given below :


(a) 1 only


(b) 2 and 3 only


(c) 1 and 3 only


(d) 1, 2 and 3

4.2k views

63. During a thunderstorm, the thunder in the skies is produced by the

  1. meeting of cumulonimbus clouds in the sky
  2. lightning that separates the nimbus clouds
  3. violent upward movement of air and water particles

Select the correct answer using the codes given below.


(a) 1 only


(b) 2 and 3


(C) 1 and 3


(d) None of the above produces the thunder

4.1k views

64. The standard time of the following countries is ahead of or behind Greenwich Mean Time depending on whether they are East or West of the longitude passing through Greenwich. 

1.Cuba

2.Greece

3. Iraq

4. Costa Rica

5. Japan

Which one of the following sequential orders gives the correct arrangement of the countries according to their standard time from ahead to behind GMT ?

(a) 5, 3, 2, 1, 4


(b) 2, 4, 1, 3, 5


(c) 4, 1, 3, 2, 5


(d) 3, 5, 4, 1, 2

4.2k views

65. From Aceh in the far north west to Torres Strait in the east is 5000 miles, almost as far as from London to Baghdad. The archipelago has 14,000 islands, some mere equatorial rocks, others some of the largest in the world.” This description best fits: 


(a) West Indies


(b) Japan


(c) Philippines


(d) Indonesia

4.1k views

Ask some economy conceptual questions next slot so that I can ask n clear my concepts.

Have put geography in this slot. Economy in next 65-70. 

3.7k views

61. A

62. C

63. D

64. A

65. D

@singhpradyumna125831 

3.7k views

61a

62c

63a help me analyse it further.

64 5 yrs se this concept not confident I leave it everywhere.

65d



65 d

Sure. 

63. Question is what cause of thunder during a thunderstorm. NOT what causes a thunderstorm.

So what causes thunder? Thunder is caused by lightning. When a lightning bolt travels from the cloud to the ground it actually opens up a little hole in the air, called a channel. Once then light is gone the air collapses back in and creates a sound wave that we hear as thunder. The reason we see lightning before we hear thunder is because light travels faster than sound.

3.7k views

61a

62c

63a help me analyse it further.

64 5 yrs se this concept not confident I leave it everywhere.

65d



65 d

64. India is at 82.5 E. We are 5.5 hrs ahead of London. So, when it is 10 pm in India, it is 430 PM in London. 

Simply put, countries lying east of GMT are ahead in time. 

Now, for such questions, when you know where they are on the world map, it can be solved. 

3.7k views

66. In the context of Indian economy, which of the following is/are the purpose/purposes of ‘Statutory Reserve Requirements’? 


1.To enable the Central Bank to  control  the  amount of advances the banks can create


2.To make the people’s deposits with banks safe and liquid


3.To  prevent  the  commercial banks from making excessive profits


4.To force  the banks to  have sufficient vault cash to meet their day-to-day requirements


Select the correct answer using the code given below.


a)1 only


b)1 and 2 only


c)2 and 3 only


d)1,2,3 and 4

3.7k views

67. When the Reserve Bank of India reduces the Statutory Liquidity Ratio by 50 basis points, which of the following is likely to happen? 


a)India’s GDP growth rate increases drastically


b)Foreign Institutional Investors may bring more capital into our country


c)Scheduled Commercial Banks may cut their lending rates


d)It may drastically reduce the liquidity to the banking system 1 and 2 only

3.7k views

68. In the context of Indian economy, consider the following pairs: 


      Term                                        Most Appropriate description


1. Melt down                                     Fall in stock prices


2. Recession                                       Fall in growth rate


3. Slow down                                     Fall in GDP


Which of the pairs given above is/are correctly matched? 


a) 1 only


b) 2 and 3 only


c) 1 and 3 only


d) 1,2 and 3

3.7k views

69. Economic liberalisation in India started with: (2000) 


a)  substantial changes in industrial licensing policy


b) the convertibility of Indian rupee


c) doing away with procedural formalities for foreign direct investment


d) significant reduction in tax rates

3.7k views

70. Which of the following measures would result in an increase in the money supply in the economy? 


1.Purchase of government securities from the public by the Central Bank


2.Deposit of currency in commercial banks by the public


3.Borrowing by the government from the Central Bank


4.Sale of government securities to the public by the Central Bank


Select the correct answer using the codes given below:


a)1 only


b)2 and 4 only


c)1 and 3


d)2,3 and 4

3.7k views

Do I need to remember facts related to census 2011??.....lot of questions pvs yr related to census 2001.

Not all of them. But yes, some important trends and figures you should be aware of.

- decadal comparison. Grew wrt prev decade etc. 

- ratios, population, literacy etc. 

- trends for mains. Migration trends etc - helps in paper 1. 


3.7k views

Ask some economy conceptual questions next slot so that I can ask n clear my concepts.

Have put geography in this slot. Economy in next 65-70. 

Posted. 65-70. Shoot!

3.7k views

66a

67c drastically 2 option eliminated

68d meltdown not 100 percent sure..I hv to analyse recession further linking with bond yields with clarity in mind...expecting ques in mains2020.

69a

70c

66. B

67. C

68. A

69. A

70. C

3.7k views

66a

67c drastically 2 option eliminated

68d meltdown not 100 percent sure..I hv to analyse recession further linking with bond yields with clarity in mind...expecting ques in mains2020.

69a

70c

Yes, mains there may be questions wrt. business cycles. 

3.7k views

@Upsc_2020 u r my coach ...why r u not posting questions regularly??...I hv to be failproof smart n alert till 25 may in choosing what to read n what not to read.

Yup, at it, Meanwhile, while we practice prev year qs here, you should also refer to 10k questions thread. 

3.6k views

71. Consider the following statements regarding the Chakiarkoothu form of dance:  


1.It is performed by Chakiar caste


2.It cannot be traditionally witnessed by the higher caste Hindus


3.Mizhavu is the accompanying instrument


4.Its theatre form is called koothambalam


Which of these statements are correct?


(a)1, 3 and 4


(b)1, 2 and 3


(c)2, 3 and 4


(d)1, 2 and 4

3.6k views

72. With reference to Indian history, who among the following is a future Buddha, yet to come to save the world?  


(a)Avalokitesvara


(b)Lokesvara


(c)Maitreya


(d)Padmapani

3.6k views

73. With reference to soil conservation, consider the following practices: 

  1. Crop rotation
  2. Sand fences
  3. Terracing
  4. Wind breaks

Which of the above are considered appropriate methods for soil conservation in India?


(a) 1, 2 and 3 only


(b) 2 and 4 only


(c) 1, 3 and 4 only


(d) 1, 2, 3 and 4

4k views

74. With reference to agricultural soils, consider the following statements : 


1.A high content of organic matter in soil drastically reduces its water holding capacity.


2.Soil does not play any role in the Sulphur cycle.


3.Irrigation over a period of time can contribute to the salinization of some agricultural lands.


Which of the statements given above is/are correct ?


(a)1 and 2 only


(b)3 only


(c)1 and 3 only


d) 1, 2 and 3

3.6k views

75. Volcanic eruptions do not occur in : 


(a)Baltic Sea


(b)Black Sea


(c) Caribbean Sea


(d) Caspian Sea

3.6k views

68. In the context of Indian economy, consider the following pairs: 


      Term                                        Most Appropriate description


1. Melt down                                     Fall in stock prices


2. Recession                                       Fall in growth rate


3. Slow down                                     Fall in GDP


Which of the pairs given above is/are correctly matched? 


a) 1 only


b) 2 and 3 only


c) 1 and 3 only


d) 1,2 and 3

I had option D in mind,as recession is defined by negative growth rate for two consecutive quarters,i.e.fall in growth rate.

could you please clear my confusion?

Because slowdown is not a fall in GDP. It is option 3 which is wrong, thus you are left with 1 & 2 - which is not an option. So the most appropriate answer is 1 only. 

Recession - you are right. 

3.6k views

Maitreya buddha question I did correct in 2017 due to random search or wiki during last 10 days before exam...does that mean to continue with such random searches till exam comprising books revision?

I would suggestnotto devote a large part of your day to random searches. Because there's no end it. You must be having a schedule right ? Whenever you're on forum, do the surfing, refer to Factoids thread etc. All that will stay with you and help you eliminate options. 

It's essential to be thorough with basics because the serious candidates do not go wrong there. 

3.6k views

71a

72c

73d

74b

75a

@samudragupt You want to take a shot ?

3.6k views

Aap 10000 initiative me expected selective question dalte hain?

Expected ka to nai pata, just more revision. Solving more and more helps. 

3.6k views

71a

72c

73d

74b

75a


72 C

73 D

74 B

75 A

3.6k views

76. In the Parliament of India, the purpose of an adjournment motion is

a) to allow a discussion on a definite matter of urgent public importance.

b) to let opposition members collect information from the ministers.

c) to allow a reduction of specific amount in demand for grant.

d) to postpone the proceedings to check the inappropriate or violent behaviour on the part of

some members.

3.6k views

77. Consider the following statements:

1. Union Territories are not represented in the Rajya Sabha.

2. It is within the purview of the Chief Election Commissioner to adjudicate the election disputes.

3. According to the Constitution of India, the Parliament consists of the Lok Sabha and the Rajya Sabha only.

Which of the statements given above is/are correct?

a) 1 only

b) 2 and 3

c) 1 and 3

d) None

3.6k views

78. Which of the following provisions of the Constitution of India have a bearing on Education?

1. Directive Principles of State Policy

2. Rural and Urban Local Bodies

3. Fifth Schedule

4. Sixth Schedule

5. Seventh Schedule

Select the correct answer using the codes given below:

(a) 1 and 2 only

(b) 3, 4 and 5 only

(c) 1, 2 and 5 only

(d) 1, 2, 3, 4 and 5


3.6k views

79. When the annual Union Budget is not passed by the Lok Sabha,

(a) the Budget is modified and presented again.

(b) the Budget is referred to the Rajya Sabha for suggestions.

(c) the Union Finance Minister is asked to resign.

(d) the Prime Minister submits the resignation of Council of Ministers.

3.6k views

80. The authorization for the withdrawal of funds from the Consolidated Fund of India must come from

(a) The President of India

(b) The Parliament of India

(c) The Prime Minister of India

(d) The Union Finance Minister

3.6k views

For ?

3.6k views

FCsaid

76- a

77-d

78- a

79- d

80-b

@singhpradyumna125831 

76 A


77 D


78 D


79 D


80 B

3.6k views

D

3.8k views

D

3.7k views

B? Which language is this. 

3.6k views

84. Which one of the following factors is responsible for the change in the regular direction of the ocean currents in the Indian Ocean?


A) Indian Ocean is 'half an ocean'


B) Indian Ocean has Monsoon drift


C) Indian Ocean is a land-locked ocean


D) Indian Ocean has greater variation in salinity

3.6k views

85. If the earth's direction of rotation is reversed, what would be the IST when it is noon at the International Date Line?

A. 06.30 hrs


B. 05.30 hrs


C. 18.30 hrs


D. 17.30 hrs

3.6k views

FCsaid

@upsc2020 can you explain what parts of option 3,4,5 have a bearing on education in Q.78 ?

Sure. 

DPSP: To provide early childhood care and education for all children until they complete the age of six years (Article 45).

Fifth Schedule – It is aimed towards making the State responsible towards the promotion ofeducationaland economic interests of Tribals.

Sixth schedule - deals with the constitution, powers and functions of District Councils and Regional Councils in these autonomous districts. These councils also have the powers to set up and administer their system of justice and maintain administrative and welfare services in respect of land, revenue, forests,education, public health etc.

Rural and ULBs: 11th schedule, 29 functions of Panchayat : item#17:educationincluding primary and secondary schools.

7th Schedule - Lists have Education. 

3.7k views

@Upsc_2020 Could you pls fix time slot to shoot questions like eve or night?

I try for before lunch & post dinner.. fixed time slot. Let me see if I can schedule it. 

3.7k views
@Patrick_jane i thought lushai of mizo are blue mountain??  :/


Yes they too. But that doesn't match any option here. Phangwpui Peak of Mizoram is known as Blue Mountain. 

3.7k views

85. If the earth's direction of rotation is reversed, what would be the IST when it is noon at the International Date Line?

A. 06.30 hrs


B. 05.30 hrs


C. 18.30 hrs


D. 17.30 hrs

India is located to the West of IDL, considering the direction of rotation of earth . So, the time at IDL (eastern side) will be ahead of India. India is located +5:30 to prime meridian time, and IDL (east) is exactly +12:00 hrs ahead of prime meridian time.

By subtracting (12-5.30), we know that Indian time is behind IDL at around 6:30 AM.

Now, even if the direction of earth’s rotation were to change, the difference would remain the same (6:30 hrs), but now India would move ahead in time of IDL (as Sun will apparently move from West to East). So, the time at India when it is noon at IDL would be evening 18:30 hrs. 

@singhpradyumna125831 

3.8k views

85. If the earth's direction of rotation is reversed, what would be the IST when it is noon at the International Date Line?

A. 06.30 hrs


B. 05.30 hrs


C. 18.30 hrs


D. 17.30 hrs

C ?

Yes.. c ..

Explanation given above. 

3.7k views

@Upsc_2020 

Corporation related facts

National thermal power corporation has diversified into hydropower 

Damodar valley corp first multipurpose river valley project independent India 

2004

Which national highways to be memorised 2007

Such patterns relevant present trend too?

Anyways thanks for detailed explanation.

Corporation related facts - just what you come across in Current Affairs. 

National Highways - which were in news. 

Whenever you do CA or any compilation, the boxes would contain such info. 

My advice would be to stick to what's in news. 

3.5k views

91. Mycorrhizal biotechnology has been used in rehabilitating degraded sites because mycorrhiza enables the plants to

1. resist drought and increase absorptive area

2. tolerate extremes of pH.

3. resist disease infestation

Select the correct answer using the codes given below.

(a) 1 only

(b) 2 and 3

(c) 1 and 3

(d) 1, 2 and 3

3.5k views

92. Satellites used for telecommunication relay are kept in a geostationary orbit. A satellite is said to be in

such an orbit when:

1. The orbit is geosynchronous

2. The orbit is circular

3. The orbit lies in the plane of the Earth’s equator

4. The orbit is at an altitude of 22,236 km.

Select the correct answer using the codes given below:

(a) 1, 2 and 3 only

(b) 1, 3 and 4 only

(c) 2 and 4 only

(d) 1, 2, 3 and 4


3.5k views

93. A company marketing food products advertises that its do not contain trans-fats. What does this

campaign signify to the customers?

1. The food products are not made out of hydrogenated oils

2. The food products are not made out of animal fats/oils

3. The oils used are not likely to damage the cardiovascular health of the consumers

Which of the statements given above is/are correct?

(a) 1 only

(b) 2 and 3 only

(c) 1 and 3 only

(d) 1, 2 and 3

3.5k views

94. Scientists can determine the arrangement or relative positions of genus of DNA sequences on a chromosome. How does this knowledge benefit us?

1. It is possible to know the pedigree of livestock.

2. It is possible to understand the causes of all human diseases.

3. It is possible to develop disease-resistant animal breeds.

Which of the statements given above is/are correct?

(a) 1 and 2 only

(b) 2 only

(c) 1 and 3 only

(d) 1, 2 and 3

3.5k views

95. Consider the following statements:

1. Chikmagalur is well-known for sugar production.

2. Mandya is well-known as a coffee-producing region.

Which of the statements given above is/are correct?

A. 1 only

B. 2 only

C. Both 1and 2

D. Neither 1 nor 2

3.5k views

Sci tech ques next round

91-94. S&T is ever evolving. PYQ very unlikely to be asked. General science questions ate repeated.

95 - Geo 

3.5k views

@singhpradyumna125831 Check 10k questions thread also please. Have put some S&T questions. 

3.5k views

96. 

3.4k views

97. 

3.4k views

98. 

3.4k views

99. 

3.4k views

100. 

3.4k views

96. D

97. X - UPSC hasn't given an answer in official key. 

98. C

99. C

100. A

3.4k views

100. 

A? 

what is the sea salinity and temperature conditions?

Sunlight: Corals need to grow in shallow water where sunlight can reach them. Corals depend on the zooxanthellae (algae) that grow inside of them for oxygen and other things, and since these algae needs sunlight to survive, corals also need sunlight to survive. Corals rarely develop in water deeper than 165 feet (50 meters).

Clear water: Corals need clear water that lets sunlight through; they don’t thrive well when the water is opaque. Sediment and plankton can cloud water, which decreases the amount of sunlight that reaches the zooxanthellae.

Warm water temperature: Reef-building corals require warm water conditions to survive. Different corals living in different regions can withstand various temperature fluctuations. However, corals generally live in water temperatures of68–90° F or 20–32° C.

Salinity: Corals occur in a salinity range between32 and 40PSU (practical salinity units). Normal sea salinity is around 35 PSU. 



3.4k views

101. 

3.4k views

102. 

3.4k views

103. 

3.4k views

101 C

102 A

103 C

@singhpradyumna125831 

3.5k views

101 c in thunderstorms advection what I can recall.... Advection (inversion )dint came into picture at all.pls give direction to my thinking on this topic.

102 a

103 d tell me which all gases come out of diesel engine...no2 so2 co..then?

101 - Your thought process is right. Advection is horizontal movement of air/currents. Even if you can recollect the diagram, you would remember, it is intense convection that takes place. 

3.5k views

101 c in thunderstorms advection what I can recall.... Advection (inversion )dint came into picture at all.pls give direction to my thinking on this topic.

102 a

103 d tell me which all gases come out of diesel engine...no2 so2 co..then?

103 Diesel fuel contains more energy per litre than petrol and coupled with the fact that diesel engines are more efficient than petrol engines, diesel cars are more efficient to run. Emissions of the regulated pollutants (carbon monoxide, hydrocarbons and nitrogen oxides) arelowerthan those from petrol cars.

3.5k views

104. 

3.5k views

105. 

3.5k views

106. 

3.5k views

107. 

3.5k views

99. 

Can anyone help me with a brief explanation for this one? TIA

Answer is C.

The Constitution confers the rights and privileges under Article14 on the citizens of India and to aliens. This explains statement 1 and 3.

Indian citizens living abroad are protected by the laws of the land of the country they are in. Essentially, the NRIs. 

All citizens born in India is a gibberish statement meant to be eliminated. 

6.3k views
» show previous quotes

Answer is C.

The Constitution confers the rights and privileges under Article14 on the citizens of India and to aliens. This explains statement 1 and 3.

Indian citizens living abroad are protected by the laws of the land of the country they are in. Essentially, the NRIs. 

All citizens born in India is a gibberish statement meant to be eliminated. 

I am not convinced why " all citizens born in India " is a gibberish statement , it is not wrong per se .
Also Article 14 does apply to Indians living abroad too , why would their protection ( if it happens) by foreign laws make this statement false? 

Can anyone please explain 

Article 14 does apply to Indians living abroad - As per my understanding, it is for people within Indian territory. 

4.5k views
@upsc2020 I think they are covered , but it may be a grey area 
"
  • NRIs are Indian citizens. They have the same benefits as all Indian Citizens barring a few exceptions.
  • They are given all the Fundamental Rights in the Constitution in the same manner as they are given to an ordinary citizen. They have voting rights, right to contest an election, avail all Fundamental Rights given in the Constitution, etc."


https://nrilegalconsultants.in/can-foreigner-nri-oci-claim-fundamental-rights/




I presume this is a grey area..and the answer I shared is from official key..

4.5k views

I think closest to Imperial preference is D. 

4.5k views

C?

3.6k views

A

Brtish discriminated/discouraged Indian products by imposing extra dutys on exports 

At same time it gave preferential treatment to brtsh imported products which is termed as imperial preference .

Its D 

I think even the official key has given D, right ?

3.6k views

What is the current situation about National anthem and FRs?

It is said to be violative of FR under Article 19 not religion. 

Current situation: It's a fundamental duty which is non enforceable.https://www.jurist.org/news/2018/01/india-supreme-court-rules-national-anthem-no-longer-required-at-cinema/

The Prevention of Insults to National Honour Act, 1971 is applicable to person who intentionally prevents the singing of the Indian National Anthem or causes disturbance to any assembly engaged in such singing.

3.9k views

What is the current situation about National anthem and FRs?

It is said to be violative of FR under Article 19 not religion. 

Current situation: It's a fundamental duty which is non enforceable.https://www.jurist.org/news/2018/01/india-supreme-court-rules-national-anthem-no-longer-required-at-cinema/

The Prevention of Insults to National Honour Act, 1971 is applicable to person who intentionally prevents the singing of the Indian National Anthem or causes disturbance to any assembly engaged in such singing.

Answer was given All three are correct(C)

I think Art 25 (Freedom of conscience, practice and propagate the religion ) has nothing to do with National anthem. This is PYQ. 

Article 25 doesn't have anything to do with it. 

3.9k views
» show previous quotes

No it is used by sc in Emmanuel case of 1986

The Supreme Court of India found that the expulsion of school children for not singing the national anthem constituted a violation of their right to freedom of expression. Three school children were expelled from school after refusing to sing the Indian national anthem since it was against their religious faith as Jehovah’s Witnesses. Their representative argued that the expulsion was an infringement of their fundamental rights to freedom expression under Article 19 and freedom of religion under Article 25 of the Constitution of India. The Court reasoned that a limitation on the right to freedom of expression must be based on a law with statutory force. Yet, there were no provisions of the law that obligated individuals to sing the national anthem and the State of Kerala’s Department of Education lacked statutory force to require school children to participate.

https://indiankanoon.org/doc/1508089/



This won't be applicable in the current situation right, post the ruling in 2018. https://www.jurist.org/news/2018/01/india-supreme-court-rules-national-anthem-no-longer-required-at-cinema/

3.9k views

120. 

3.9k views

121. 

3.9k views

122. 

3.9k views

120. D

121. A

122. B


(acc. to official key of UPSC)

3.9k views

123.

3.9k views

124. 

3.9k views

125. 

3.6k views

126. 

3.6k views

123 A

124 C

125 C

126 C

3.5k views

127. 

3.5k views

128

3.5k views

129. 

3.5k views

130. 

3.5k views

131. 

3.5k views

127 D

128 B

129 B

130 B

131 B



3.4k views

@upsc2020  it would be even more helpful if you can mention the year in which the question appeared along with the question. 

Regards

Sure.

3.4k views

125c

126c

127d

128b.. Mahabalipuram in news too.

129b

130b

131a..lack clarity.

131 any particular concept?

3.4k views

132. 

CAPF 16

3.4k views

133. 

3.4k views

134. 

3.4k views

132 A .

133 A

134 D Salt in the ocean comes from two sources: runoff from the land and openings in the seafloor.

3.8k views

131. 

Crowding out effect refers to how increased government spending, for which it must borrow more money, tends to reduce private spending. This happens because when the government takes up the major chunk of funds available in the banking system, less of it is left for private borrowers.

Crowding in – this relates to how higher government spending encourages firms to invest more.This is due to the income effect of higher government spending. If the economy is in a recession or below full capacity, expansionary fiscal policy can increase the economic growth rate and create a positive multiplier effect, which leads to greater private sector investment.


The Question has been directly framed from Economic Survey of 2017. 

https://www.thehindu.com/news/national/other-states/survey-backs-up-model-on-farm-loan-waivers/article19476458.ece


@AlexanderSupertramp 

@singhpradyumna125831 

@Greenlove

6.6k views

131. 

Crowding out effect refers to how increased government spending, for which it must borrow more money, tends to reduce private spending. This happens because when the government takes up the major chunk of funds available in the banking system, less of it is left for private borrowers.

Crowding in – this relates to how higher government spending encourages firms to invest more.This is due to the income effect of higher government spending. If the economy is in a recession or below full capacity, expansionary fiscal policy can increase the economic growth rate and create a positive multiplier effect, which leads to greater private sector investment.


The Question has been directly framed from Economic Survey of 2017. 

https://www.thehindu.com/news/national/other-states/survey-backs-up-model-on-farm-loan-waivers/article19476458.ece


@AlexanderSupertramp 

@singhpradyumna125831 

@Greenlove 

3.8k views
135.Consider the following pairs

Sea: Bordering country

1) Adriatic Sea: Albania

2) Black Sea: Croatia

3) Caspian Sea: Kazakhstan

4) Mediterranean Sea: Morocco

5) Red Sea: Syria

Which of the pairs given above are correctly matched?

A. 1, 2 and 4 
B. 1, 3 and 4 
C. 2 and 5 
D. 1, 2, 3, 4,5
3.7k views
136. What is the correct sequence of occurrence of the following cities in South-East Asia as one proceeds from south to north?

1. Bangkok 
2. Hanoi
3.Jakarta
4. Singapore

Select the correct answer using the code given below.

A. 4-2-1-3
B. 3-2-4-1
C. 3-4-1-2
D. 4-3-2-1

Cse 14
3.7k views

137. 

3.7k views
138. Consider the following statements:

1. The Axis of the earth's magnetic field is inclined at 23 and half to the geographic axis of the earth.

2. The earth's magnetic pole in the Northern Hemisphere is located on a Peninsula in Northern Canada.

3. The earth's magnetic equator passes through Thumba in South India.

Which of the statements given above is/are correct?

A. 1, 2 and 3
B. 2 and 3 
C. 2 only
D. 3 only
3.7k views

135. B

136. A

137. Bgeometry and some characteristics of the magnetic field is very similar to one that would be produced, if a large bar magnet (a dipole magnet - having two poles: north and south) were located in the center of the Earth, inclined (about 11 degrees at present) from the rotational axis of the Earth.

3.6k views
138. With reference to the legislative Assembly of a state in India, consider the following Statements:

1) The Governor makes a customary address to Members of the house at the commencement of the first session of the year.

Select correct answer. 
A. 1 only 
B. 2 only 
C. Both 1 and 2 
D. Neither 1 nor 2
2) When a State Legislature does not have a rule on the particular matter, it follows the lok Sabha rule on that matter.
3.6k views
138. With reference to the legislative Assembly of a state in India, consider the following Statements:

1) The Governor makes a customary address to Members of the house at the commencement of the first session of the year.

Select correct answer. 
A. 1 only 
B. 2 only 
C. Both 1 and 2 
D. Neither 1 nor 2
2) When a State Legislature does not have a rule on the particular matter, it follows the lok Sabha rule on that matter.

a


state govt follows provincial assembly rules if there are no precedents

That's correct.

3.6k views
139. What will follow if the money bill is substantially amended by the Rajya Sabha?

(a) The Lok Sabha may still proceed with the bill, accepting or not accepting the recommendations of the Rajya Sabha
(b) The Lok Sabha cannot consider the bill further
(c) The Lok Sabha may send the bill to the Rajya Sabha for reconsideration
(d) The president may call a joint sitting for passing the Bill
3.5k views
140. With reference to the history of Indian rock-cut architecture, consider the following statements:

1. The caves at Badami are the oldest surviving rock-cut caves in India
2. The Barabar rock-cut caves were originally made for Ajivikas by Emperor Chandragupta Maurya
3. At Ellora, caves were made for different faiths.
Which of the following statements given above is/are correct?

(a) 1 only
(b) 2 and 3 only
(c) 3 only
(d) 1, 2 and 3
3.5k views
141. Consider the following :

1. Star tortoise
2. Monitor lizard
3. Pygmy hog
4. Spider monkey
Which of the above are naturally found in India?

(a) 1, 2 and 3 only
(b) 2 and 3 only
(c) 1 and 4 only
(d) 1, 2, 3 and 4
3.5k views
142. Consider the following animals:

1. Sea cow
2. Sea horse
3. Sea lion
Which of the above is/are mammal/mammals?

(a) 1 only
(b) 1 and 3 only
(c) 2 and 3 only
(d) 1, 2 and 3
3.5k views
143.  Consider the following :

1. Electromagnetic radiation
2. Geothermal energy
3. Gravitational force
4. Plate movements
5. Rotation of the earth
6. Revolution of the earth
Which of the above are responsible for bringing dynamic changes on the surface of the earth?

(a) 1, 2, 3 and 4 only
(b) 1, 3, 5 and 6 only
(c) 2, 4, 5 and 6 only
(d) 1, 2, 3, 4, 5 and 6
3.5k views
144. Improper handling and storage of cereal grains and oilseeds result in the production of toxins known as aflatoxins which are not generally destroyed by normal cooking process. Aflatoxins are produced by 

(a) bacteria
(b) protozoa
(c) moulds
(d) viruses
3.5k views
145. With reference to food chain in ecosystems, consider the following statements:

1. A food chain illustrates the order in which a chain of organisms feed upon each other
2. Food chains are found within the populations of a species
3. A food chain illustrates the number of each organism which are eaten by others.
Which if the statement given above is/are correct?

(a) 1 only
(b) 1 and 2 only
(c) 1, 2 and 3
(d) None
3.5k views

139 A

140 C

141 A

142 B

143 D

144 C

145 A

@AlexanderSupertramp

3.4k views
143.  Consider the following :

1. Electromagnetic radiation
2. Geothermal energy
3. Gravitational force
4. Plate movements
5. Rotation of the earth
6. Revolution of the earth
Which of the above are responsible for bringing dynamic changes on the surface of the earth?

(a) 1, 2, 3 and 4 only
(b) 1, 3, 5 and 6 only
(c) 2, 4, 5 and 6 only
(d) 1, 2, 3, 4, 5 and 6

Ans- a


3- mass movement of debris, mass wasting

4- earthquakes


5,6- no dynamic change as such, just days and seasons


According to Official Key of UPSC, answer is D. 

3.4k views
141. Consider the following :

1. Star tortoise
2. Monitor lizard
3. Pygmy hog
4. Spider monkey
Which of the above are naturally found in India?

(a) 1, 2 and 3 only
(b) 2 and 3 only
(c) 1 and 4 only
(d) 1, 2, 3 and 4

ans- a?


1- yes

2- yes, goa heritage instrument gumot made from it

3- yes, in manas national park

4- no idea 

Spider monkey lives in forests from southern Mexico through Central and South America to Brazil. According to the International Union for Conservation of Nature (IUCN) Red List of Threatened Species, all true spider monkey species are threatened. 

3.4k views
143.  Consider the following :

1. Electromagnetic radiation
2. Geothermal energy
3. Gravitational force
4. Plate movements
5. Rotation of the earth
6. Revolution of the earth
Which of the above are responsible for bringing dynamic changes on the surface of the earth?

(a) 1, 2, 3 and 4 only
(b) 1, 3, 5 and 6 only
(c) 2, 4, 5 and 6 only
(d) 1, 2, 3, 4, 5 and 6

Ans- a


3- mass movement of debris, mass wasting

4- earthquakes


5,6- no dynamic change as such, just days and seasons


According to Official Key of UPSC, answer is D. 

so, seasons and day night changes as considered dynamic changes too

Correct...

3.4k views
146. Which of the statements given below are correct?

1. A person having blood group 'A' can donate blood to persons having blood group 'A' and blood group 'AB'.

2. A person having blood group 'AB' can donate blood to persons having blood groups 'A', 'B', 'AB' or '0'.

3. A person with blood group 40' can donate blood to persons having any blood group.

4. A person with blood group '0' can receive blood from the person of any of the blood groups.

Select the correct answer using the code given below

(a) 1, 2, 3 and 4

(b) 1 and 2 only

(c) 3 and 4 only

(d) 1 and 3 only

3.4k views
147. Which one of the following types of pesticides is convenient to control stored grain pests?

(a) Systemic pesticides

(b) Fumigants

(c) Contact poisons

(d) Stomach poisons
3.4k views
148. Which one of the following types of pesticides is convenient to control stored grain pests?

(a) Systemic pesticides

(b) Fumigants

(c) Contact poisons

(d) Stomach poisons
3.4k views
149.  Arrange the following tiger reserves of India from North to South :

1. Indravati

2. Dudhwa

3. Bandipur

4. Similipal

Select the correct answer using the code given below :

(a) 3 - 4 - 1 - 2

(b) 4 - 2 - 3 - 1

(c) 2 - 4 - 1 -3

(d) 2 - 1 - 4 - 3

3.4k views
150. Consider the following statements about State Election Commission:

1. The State Election Commissioner shall be appointed by the Governor of the State.

2. The State Election Commission shall have the power of even preparing the electoral rolls besides the power of superintendence, direction .and control of election to the panchayats.

3. The State Election Commissioner cannot be removed in any manner from his office until he demits himself or completes his tenure.

Which of the above statements is/are correct?

(a) 1, 2 and 3

(b) 1 and 2 only

(c) 2 and 3 only

(d) 1 only
3.4k views
149.  Arrange the following tiger reserves of India from North to South :

1. Indravati

2. Dudhwa

3. Bandipur

4. Similipal

Select the correct answer using the code given below :

(a) 3 - 4 - 1 - 2

(b) 4 - 2 - 3 - 1

(c) 2 - 4 - 1 -3

(d) 2 - 1 - 4 - 3

Dudhwa- UP

Bandipur- Karnataka

Simlipal- Odisha

so the correct option should be c or d

Indravati- tributary of Godavari- passes through maharashtra,telangana etc.


I think it should be C

C Is correct. 

3.3k views

146 D

147 B

148 B

149 C

150 B

3.3k views
147. Which one of the following types of pesticides is convenient to control stored grain pests?

(a) Systemic pesticides

(b) Fumigants

(c) Contact poisons

(d) Stomach poisons


3.3k views
151. The Limitation Law, which was passed by the British in 1859, addressed which one of the following
issues?

A) Loan bonds would not have any legal validity.
B) Loan bonds signed between money -lender and Ryots would have validity only for three years.
C) land bonds could not be executed by moneylenders.
D) Loan bonds would have validity for ten years.
3.3k views

152. The Summary Settlement of 1856 was based on which one of the following assumptions?

A) The Talukdars were the rightful owners of the land.
B) The Talukdars were interlopers with no permanent stakes in the land.
C) The Talukdars could evict the peasants from the lands.
D) The Talukdars would take a portion of the revenue which flowed to the State.

3.3k views
153. Which among the following statements about the power to change the basic structure of the
Constitution of India is/are correct?

1. It falls outside the scope of the amending powers of the Parliament.
2. It can be exercised by the people through representatives in a Constituent Assembly.
3. It falls within the constituent powers of the Parliament

Select the correct answer using the code given below.
A) 1 and 3
B) 1 and 2
C) 1 only
D) 2 and 3
3.3k views
154. Which one of the following is not correct about the Panchayats as laid don in part IX of the
Constitution of India?

A) The Chairperson of a Panchayat needs to be directory elected by people in order to exercise the right
to vote in the Panchayat meetings.
B) The State Legislature has the right to decide whether or not offices of the Chairpersons in the
Panchayats are reserved for SCs, STS or women.
C) Unless dissolved earlier, every Panchayat continues for a period of five years.
D) The State Legislature may by law make provisions for audit of accounts of the Panchayats.
3.3k views

155. Which one of the following is not correct about Administrative Tribunals?

A) The parliament may by law constitute Administrative Tribunals both at the Union and State levels.
B) Tribunals may look into disputes and complaints with respect to recruitment and conditions of
service of persons appointed to public services.
C) Tribunals established by a law of the Parliament can exclude the Jurisdiction of all Courts to allow for
special leave to appeal.
D) The law establishing the Tribunals may provide for procedures including rules of evidence to be
followed.

3.4k views

156. Match List-I with List-II and select the correct answer using the code given below the Lists :

List – I              List -II
(Type of Lake) (Example)
A. Tectonic   1. Lonar lake
B. Crater       2. Gangabal Lake
C. Glacial      3. Purbasthali Lake
D. Fluvial      4. Bhimtal Lake

Code :

A) A 4 B 1 C 2 D 3
B) A 4 B 2 C 1 D 3
C) A 3 B 1 C 2 D 4
D) A 3 B 2 C 1 D 4

3.3k views
151. The Limitation Law, which was passed by the British in 1859, addressed which one of the following
issues?

A) Loan bonds would not have any legal validity.
B) Loan bonds signed between money -lender and Ryots would have validity only for three years.
C) land bonds could not be executed by moneylenders.
D) Loan bonds would have validity for ten years.

@upsc2020 which year was it asked?

2019 CDS. 

I am picking those questions from CDS and CAPF papers that have a possibility to make it to CSE in my opinion.

3.8k views
152. Which of the following is/are the feature(s) of the Brahmadeya Grants during c 600-1200 AD?

1. Their creation meant a renunciation of actual or potential sources of revenue by the State.

2. These grants could vary from a small plot to several villages.

3. Most grants were made in unsettled areas

Select the correct answer using the code given below.

a) 1 only

b) 2 and 3 only

c) 1 and 2 only

d) 1, 2 and 3
3.8k views
153. The idea of ‘Farr-I Izadi’, on which the Mughal Kingship was based, was first developed by which one of the following Sufi saints?

a) Shihabuddin Suhrawadi

b) Nizamuddin Auliya

c) Ibn al-Arabi

d) Bayazid Bistani
3.8k views
154. Which one of the following is not an exclusive right of the concerned coastal nations over Exclusive Economic Zone (EEZ)?

a) Survey and exploitation of mineral resources of ocean deposits

b) Exploitation of marine water energy and marine organisms including fishing

c) Conservation and management of marine resources

d) Navigation of ships and laying down submarine cables
3.8k views
155. A person is disqualified for being chosen as, and for being, a Member of either House of the Parliament if the person

1. holds any office of profit under Government of India or the Government of any State other than an office declared by the Parliament by law not to disqualify its holder

2. is an undischarged insolvent

3. is so disqualified under the Tenth Schedule of the Constitution of India

4. is of unsound mind and stands so declared by a competent Court

Select the correct answer using the code given below.

a) 1, 2 and 4 only

b) 1, 2, 3 and 4

c) 3 and 4 only

d) 1, 2 and 3 only
3.8k views
156. Verses ascribed to poet-saint Kabir have been compiled in which of the following traditions?

1. Bijak in Varanasi

2. Kabir Granthavali in Rajasthan

3. Adi Granth Sahib

Select the correct answer using the code given below.

a) 1 and 2 only

b) 1, 2 and 3

c) 2 and 3 only

d) 3 only
3.8k views
157. Who was/were the 10th century composer(s) of the Nalayira Divya Prabandham?

a) Alvars

b) Nayanars

c) Appar

d) Sarnbandar
3.8k views

M nt getting bond yields recession relation ship.


Any question ? Can you please elaborate the doubt ?

3.7k views
@upsc2020 can you please explain this? Why Adi Granth Sahib is not correct? Ans to 156 should it not be 1,2,3?


All three are correct. 

Verses ascribed to Kabir have been compiled in three distinct but overlapping traditions. The Kabir Bijak is preserved by the Kabirpanth (the path or sect of Kabir) in Varanasi and elsewhere in Uttar Pradesh; the Kabir Granthavali is associated with the Dadupanth in Rajasthan, and many of his compositions are found in the Adi Granth Sahib. 

Ref: NCERT

3.7k views
157. Who was/were the 10th century composer(s) of the Nalayira Divya Prabandham?

a) Alvars

b) Nayanars

c) Appar

d) Sarnbandar

Nalayira Divya Prabandham is a collection of 4,000 Tamil verses composed by the 12 Alvars and was compiled in its present form by Nathamunigal during the 9th -10th centuries. These were sung by the Alvars in devotion of Vishnu.

3.7k views
155. A person is disqualified for being chosen as, and for being, a Member of either House of the Parliament if the person

1. holds any office of profit under Government of India or the Government of any State other than an office declared by the Parliament by law not to disqualify its holder

2. is an undischarged insolvent

3. is so disqualified under the Tenth Schedule of the Constitution of India

4. is of unsound mind and stands so declared by a competent Court

Select the correct answer using the code given below.

a) 1, 2 and 4 only

b) 1, 2, 3 and 4

c) 3 and 4 only

d) 1, 2 and 3 only

All correct.


Article 102.

A person shall be disqualified for being chosen as, and for being, a member of either House of Parliament —

(a) if he holds any office of profit under the Government of India or the Government of any State, other than an office declared by Parliament by law not to disqualify its holder;

(b) if he is of unsound mind and stands so declared by a competent court;

(c) if he is an undischarged insolvent;

(d) if he is not a citizen of India, or has voluntarily acquired the citizenship of a foreign State, or is under any acknowledgment of allegiance or adherence to a foreign State;

(e) if he is so disqualified by or under any law made by Parliament.

(2) A person shall be disqualified for being a member of either House of Parliament if he is so disqualified under the Tenth Schedule.

3.7k views
154. Which one of the following is not an exclusive right of the concerned coastal nations over Exclusive Economic Zone (EEZ)?

a) Survey and exploitation of mineral resources of ocean deposits

b) Exploitation of marine water energy and marine organisms including fishing

c) Conservation and management of marine resources

d) Navigation of ships and laying down submarine cables

In the exclusive economic zone, the coastal nation has sovereign rights for the purpose of exploring and exploiting, conserving and managing the natural resources, whether living or non-living, including production of energy from the water, currents and winds. That means first three rights are given to coastal states.

However, In the exclusive economic zone, all nations, whether coastal or land-locked shall enjoy the freedom of navigation and overflight and of the laying of submarine cables and pipelines, and other internationally lawful uses of the sea. That means Navigation of ships and laying down submarine cables  is not exclusive only for the coastal nations.

3.7k views
153. The idea of ‘Farr-I Izadi’, on which the Mughal Kingship was based, was first developed by which one of the following Sufi saints?

a) Shihabuddin Suhrawadi

b) Nizamuddin Auliya

c) Ibn al-Arabi

d) Bayazid Bistani

Abul Fazl placed Mughal kingship as the highest station in the hierarchy of objects receiving light emanating from God (farr-i izadi). Abul Fazl was inspired by a famous Iranian Sufi, Shihabuddin Suhrawardi (d.1191) who first developed this idea. According to this idea, there was a hierarchy in which the Divine Light was transmitted to the king who then became the source of spiritual guidance for his subjects.



3.7k views
152. Which of the following is/are the feature(s) of the Brahmadeya Grants during c 600-1200 AD?

1. Their creation meant a renunciation of actual or potential sources of revenue by the State.

2. These grants could vary from a small plot to several villages.

3. Most grants were made in unsettled areas

Select the correct answer using the code given below.

a) 1 only

b) 2 and 3 only

c) 1 and 2 only

d) 1, 2 and 3

D is correct. 

Brahmadeya (given to Brahmin) was tax free land gift either in form of single plot or whole villages donated to Brahmans in the early medieval(600-1200 AD) India. As per many inscriptions like Uttarmerur during Chola large scale land was expanded under agriculture and irrigation. 

4.1k views
161.Consider the following countries: 

1. Albania
2. Bosnia Herzegovina 
3. Croatia
4. Macedonia

Which of these countries has/have Adriatic Sea as a boundary?
A. 1,2,4 
B.2, 3
C.1,2,3
D. 1,2,3,4
3.3k views

162. 

3.3k views

163. 

3.3k views

163. 

B

@Anjali8 

3.4k views

162. 

A

@Anjali8 

3.4k views

174)

A


4.1k views
Can we start with this once again? It was quite useful. 

Surely. 

3.5k views

CSE20said

CSE20said

CSE20said

When the Chief Justice of a High Court acts in administrative capacity, he is subject to 

a. the writ jurisdiction of any of the other judges of the High Court

b. Special control exercised by the CJI 

c. Discretionary powers of the Governor of the State

d. Special powers provided to the CM in this regard.

A

Correct option C.

Article 227 (3) subject to any law in force and prior approval of the Governor. 

Nah. Answer is A.  


Okay I am confused now🙂 which answer key to believe? Checked with Laxmikant and it says correct answer is B!!

However Article 227(3) nowhere mentions the word "discretion"of governor, just says "prior approval of the governor". Are the two expressions one and the same thing?


Prior approval is not same as discretion. 

3.7k views

CSE20said

CSE20said

CSE20said

CSE20said

When the Chief Justice of a High Court acts in administrative capacity, he is subject to 

a. the writ jurisdiction of any of the other judges of the High Court

b. Special control exercised by the CJI 

c. Discretionary powers of the Governor of the State

d. Special powers provided to the CM in this regard.

A

Correct option C.

Article 227 (3) subject to any law in force and prior approval of the Governor. 

Nah. Answer is A.  


Okay I am confused now🙂 which answer key to believe? Checked with Laxmikant and it says correct answer is B!!

However Article 227(3) nowhere mentions the word "discretion"of governor, just says "prior approval of the governor". Are the two expressions one and the same thing?


Prior approval is not same as discretion. 

Yeah. So what do you think should be the correct answer? 

I would go with A. 

4.4k views

Agogsaid

@upsc2020 How r u guys going on with minor rivers and places lik Chandrabhaga,Pandharpur,Aliyar,Kangsabati??And even minor ranges lik Barashigri??

Only with respect to important National Parks and reserves. Or if it has been in news.  

4k views

Agogsaid

@upsc2020 lik any river passing through national park even if minor u google??


No, I have jotted down the name of the river next to my notes for the Reserves and parks. I do not try and find any other info beyond the name. Attached pic in comment below.

3.9k views

Agogsaid

@upsc2020 lik any river passing through national park even if minor u google??


No, I have jotted down the name of the river next to my notes for the Reserves and parks. I do not try and find any other info beyond the name. 


6.5k views

Agogsaid

@upsc2020 Pls do provide a list of imp ranges if any.


Haven't made any such list ya, sorry. But I can list it for you region wise if that helps. For physical Geo I haven't gone beyond Majid Hussain. It is more than sufficient. 

3.9k views
If a wetland of international importance is brought under the ‘Montreux Record’, what does it imply? 



(a) Changes in ecological character have occurred, are occurring or are likely to occur in the wetland as a result of human interference

(b) The country in which the wetland is located should enact a law to prohibit any human activity within five kilometres from the edge of the wetland

(c) The survival of the wetland depends on the cultural practices and traditions
of certain communities living in its vicinity and therefore the cultural diversity therein should not be destroyed

(d) It is given the status of ‘World Heritage Site’
3.6k views

201. 

3.6k views

202. 

3.6k views

203. 

3.6k views

204. 

3.6k views

205. 

3.6k views

205. 

which year?

CDS 2020

6.1k views

205. 

which year?

A - Rohri Hills 

Rohri Hills in Sindh: Large Indus chert workshop has been found here. 

3.4k views

204. 

this is of cse or any other exam?

D

Landform (Geomorphic) factors influencing plant and animal distributions include:

(1) Slope angle: steepness of the slope. This affects the rate at which precipitation drains from a surface; Steep slopes = rapid runoff, thicker soil; Gentle slopes = more precipitation penetrates into the soil, providing a moister habitat

(2) Slope Aspect: the orientation of a sloping ground surface with respect to geographic north;

(3)Relief: the difference in elevation of divides and adjacent valley bottoms

3.6k views

205. 

A


correct..

3.6k views

202. 

A

Correct. 

3.6k views

201. 

B, C, D are factually correct. Thus, by elimination, A should be wrong. 

But, I don't see anything wrong with the statement. Can anyone explain ?

Legislature of a state may authorize a Panchayat to levy, collect and appropriate taxes, duties, tolls and fees. The statement indicates it's a suo moto action by Panchayat which it is not. 

3.7k views
206. 
3.5k views
207. 
3.5k views

208. The formulation of policy in respect to Intellectual Property Rights (IPRs) is the responsibility of

(a) the Ministry of Law and Justice
(b) the Department of Science and Technology
(c) the Department for Promotion of Industry and Internal Trade
(d) the Ministry of Human Resource Development


3.5k views
209. The provisions of the Constitution of India pertaining to the institution of Panchayat do not apply to which one of the following States?
(a) Meghalaya 
(b)Tripura
(c) Assam
(d) Goa
3.5k views
209. The provisions of the Constitution of India pertaining to the institution of Panchayat do not apply to which one of the following States?
(a) Meghalaya 
(b)Tripura
(c) Assam
(d) Goa

A

Correct. 

3.6k views
207. 

C. Art 32 provides only for enforcement of Part III rights. Otherwise HC is the right forum 

That's correct. For SC, 32 extends to FR. 

3.6k views
207. 
3.6k views

201. 

B, C, D are factually correct. Thus, by elimination, A should be wrong. 

But, I don't see anything wrong with the statement. Can anyone explain ?

Option B (243H:Authorise Panchayat to levy .collect, appropriate such taxes ,duties .....)

Option A is correct.

3.4k views

210.

3.4k views

211. 

3.4k views

212. 

3.4k views

213. 

3.4k views

214. 

3.4k views

215. 

3.4k views

216. 

3.4k views

217. 

3.4k views

218. 

3.4k views

Hi request you to please reply to the posts or put the Question number. Can't decipher it is an answer to which Question

3.9k views

210.

A

3.7k views

211. 

C

Monopolistic competition is a market structure which combines elements of monopoly and competitive markets. A monopolistic competitive industry has the following features:

  • Many firms.
  • Freedom of entry and exit.
  • Firms produce differentiated products.
  • Firms have price inelastic demand; they are price makers because the good is highly differentiated
  • Firms make normal profits in the long run but could make supernormal profits in the short term
  • Firms are allocatively and productively inefficient
3.7k views

213. 

B

The  Chairperson of a Panchayat and other members of a  Panchayat whether   or   not  chosen  by   direct   election   from   territorial constituencies  in the Panchayat area shall have the right to vote  in the meetings of the Panchayats.


3.7k views

214. 

C

 One third shall be elected by electorates consisting of members of municipalities, district boards and such other local authorities in the State as Parliament may by law specify.

3.7k views

215. 

B

3.7k views

216. 

A

Mataji Maharani Tapaswini was one of the strongest proponents of female education in India. Her greatest contribution came in the form of the Mahakali Pathshala which she set up in Kolkata in 1893.

The school was a completely indigenous affair which did not rely on either foreign aid or assistance. The education of girls was carried out on a strictly national basis in the hopes that they would be able to revive and regenerate Hindu society.

3.7k views

218. 

B? Damn nice question. From what I've read, the basic structure of the constitution can be changed in the following ways: 

1. Formation of a new Constituent Assembly by the sovereign power(the people) to replace the old Constitution 

2. By revolution resulting in overthrow of the Republic established by the Constitution of 1950

The Parliament is merely a delegate of power under the Constitution, and therefore, it does not have the authority to destroy the fundamental features of the very document that: 1. Establishes it; 2. allows and legitimises the usage of legislative/constituent powers exercised by it. (Based on reading of Keshavanand Bharti and Minerva Mills)

Don't know which answer UPSC will accept as correct though. 

B is the official answer ;)

3.6k views
219. 
3.5k views
220. 
3.5k views
221. 
4.1k views
222. 
4.1k views

msdiansaid

» show previous quotes

@Upsc_2020 Sorry for not quoting in my earlier message. New here.

I am confused which statement is wrong? A or B. 

State legislatures only have a say on the quantum of members and Chairpersons for SCs and STs while the reservation for Women as members and chairperson is Minimum 33%. 
In the first statement Chairperson by default has the right to vote in the Panchayat meetings or are there any exceptions.
Thanks for your initiative.

The question is asking which statement isNOTcorrect. Now, let's check each statement according to the constitution.

D is correct- 243 J. Audit of accounts of Panchayats.- The Legislature of a State may, by law, make provisions with respect to the maintenance of accounts by the Panchayats and the auditing of such accounts.

C is correct- 243 E . Duration of Panchayats, etc.- Every Panchayat, unless sooner dissolved under any law for the time being in force, shall continue for five years from the date appointed for its first meeting and no longer.

B is correct - 243 D The offices of the Chairpersons in the Panchayats at the village or any other level shall be reserved for the Scheduled Castes, the Scheduled Tribes and women in such manner as the Legislature of a State may, by law, provide: This means it is the prerogative of the state legislature to decide THE MANNER, not just quantum. 

A is incorrect - 243 C The Chairperson of a Panchayat and other members of a Panchayat whether or not chosen by direct election from territorial constituencies in the Panchayat area shall have the right to vote in the meetings of the Panchayats.

4.1k views
222. 

B I guess. Statement 1 is unrelated, and Statement 4 might not be necessarily true as other minor forms of irrigation may be there. 

That's correct. 

@mhs11 

3.8k views
219. 

A?

Correct. 

3.8k views

mhs11said

221. 

A

Correct. 

3.8k views

Agogsaid

@upsc2020 frm wher 2 read abt these lakes?Never hrd!!


You just have to know Bhimtal and Lonar. That's available in several texts. 

One has to learn to eliminate for pre. We can't know everything. :)

3.8k views
@upsc2020 Quite helpful and thank you for the initiative. Went through all the questions posted here. Could you post the answer for Q152, which is about the summary settlement of 1856.
Also, there was one question about the misnomer of first, second and third round table conferences and why are these called so? There was no answer to this one, could you please check that? 


Hi can u please repost that Question?

3.4k views
220. Which of the following is/are the feature(s) of the Brahmadeya Grants during c 600-1200 AD?

1. Their creation meant a renunciation of actual or potential sources of revenue by the State.

2. These grants could vary from a small plot to several villages.

3. Most grants were made in unsettled areas

Select the correct answer using the code given below.

(a) 1 only

(b) 2 and 3 only

(c) 1 and 2 only

(d) 1, 2 and 
3.3k views
221. Which of the following statements about Sir Syed Ahmad Khan is/are correct?

1. He argued that India was federation of ethnic communities based on common descent.

2. His Philosophy was very similar to that of the Indian National Congress.

3. He imagined India as a Nation State based on individual citizen’s rights.

4. The cumclusum at the Mohammedan Anglo-Oriental College blended Muslim theology and European empiricism.

Select the correct answer using the code given below.

(a) 1 only

(b) 2 and 3

(c) 3 only

(d) 1 and 4
3.3k views
222. Which one of the following elements is essential for the formation of chlorophyll in green plants?

a) Calcium

b) Iron

c) Magnesium

d) Potassium
3.8k views
223. Around twelfth century Sufi Silsilas began to crystallize in different parts of the Islamic world to signify

1. continuous link between the master and disciple

2. unbroken spiritual genealogy to the Prophet Muhammad

3. the transmission of spiritual power and blessings to devotees

Select the correct answer using the code given below.

(a) 1 and 2 only

(b) 2 only

(c) 1 and 3 only

(d) 1, 2 and 3
3.9k views
224. In the 10th Mandala of the Rigveda, which one of the following hymns reflects upon the marriage ceremonies?

(a) Surya Sukta

(b) Purusha Sukta

(c) Dana Stutis

(d) Urna Sutra
3.8k views
225. Which of the following are the functions of the National Human  Rights Commission (NHRC)?

1. Inquiry at its own initiative on the violation of human rights

2. Inquiry on a petition presented to it by a victim

3. Visit to jails to study the condition of the inmates

4. Undertaking and promoting research in the field of human rights

Select the correct answer using the code given below.

(a) 1 and 2 only

(b) 2, 3 and 4 only

(c) 1, 3 and 4 only

(d) 1, 2, 3 and 4
3.8k views

152. The Summary Settlement of 1856 was based on which one of the following assumptions?

A) The Talukdars were the rightful owners of the land.
B) The Talukdars were interlopers with no permanent stakes in the land.
C) The Talukdars could evict the peasants from the lands.
D) The Talukdars would take a portion of the revenue which flowed to the State.

@upsc2020 this one... 

B

The first British revenue settlement, known as the Summary Settlement of 1856, was based on the assumption that the taluqdars were interlopers with no permanent stakes in land: they had established their hold over land through force and fraud.

6.4k views
222. Which one of the following elements is essential for the formation of chlorophyll in green plants?

a) Calcium

b) Iron

c) Magnesium

d) Potassium

C? Mg

Elements essential for the formation of Chlorophyll in green plants is Magnesium. Without magnesium, chlorophyll cannot capture sun energy needed for photosynthesis

3.3k views

upsc2020said

Why did Buddhism started declining in India  in early medieval times ?

1. Buddha was by that time considered as one of the incarnations of Vishnu and thus became a part of Vaishnavism.

2. Invading tribes from Central Asia till the time of last Gupta king adopted Hinduism and persecuted Buddhists.

3. Kings of Gupta dynasty were strongly opposed to Buddhism.

Which of the above given statements are correct?

A. 1 only 

B. 1 and 3 only 

C. 2 and 3 only 

D. 1, 2 and 3

A?

Correct. 

3.7k views
221. Which of the following statements about Sir Syed Ahmad Khan is/are correct?

1. He argued that India was federation of ethnic communities based on common descent.

2. His Philosophy was very similar to that of the Indian National Congress.

3. He imagined India as a Nation State based on individual citizen’s rights.

4. The cumclusum at the Mohammedan Anglo-Oriental College blended Muslim theology and European empiricism.

Select the correct answer using the code given below.

(a) 1 only

(b) 2 and 3

(c) 3 only

(d) 1 and 4

option A: 1 only

D is correct.

3.7k views
223. Around twelfth century Sufi Silsilas began to crystallize in different parts of the Islamic world to signify

1. continuous link between the master and disciple

2. unbroken spiritual genealogy to the Prophet Muhammad

3. the transmission of spiritual power and blessings to devotees

Select the correct answer using the code given below.

(a) 1 and 2 only

(b) 2 only

(c) 1 and 3 only

(d) 1, 2 and 3

D?

Correct. 

3.7k views
225. Which of the following are the functions of the National Human  Rights Commission (NHRC)?

1. Inquiry at its own initiative on the violation of human rights

2. Inquiry on a petition presented to it by a victim

3. Visit to jails to study the condition of the inmates

4. Undertaking and promoting research in the field of human rights

Select the correct answer using the code given below.

(a) 1 and 2 only

(b) 2, 3 and 4 only

(c) 1, 3 and 4 only

(d) 1, 2, 3 and 4

D

Correct. 

The Commission shall, perform all or any of the following functions, namely:-

a) Inquire, on its own initiative or on a petition presented to it by a victim or any person on his behalf, into complaint of-

i ) violation of human rights or abetment or

ii) negligence in the prevention of such violation, by a public servant;

b) intervene in any proceeding involving any allegation of violation of human rights pending before a court with the approval of such court;

c) visit, under intimation to the State Government, any jail or any other institution under the control of the State Government, where persons are detained or lodged for purposes of treatment, reformation or protection to study the living condition of the inmates and make recommendations thereon ;

d) review the safeguards by or under the Constitution or any law for the time being in force for the protection of human rights and recommend measures for their effective implementation;

e) review the factors, including acts of terrorism that inhibit the enjoyment of human rights and recommend appropriate remedial measures;

f) study treaties and other international instruments on human rights and make recommendations for their effective implementation;

g) undertake and promote research in the field of human rights;

h) spread human rights literacy among various sections of society and promote awareness of the safeguards available for the protection of these rights through publications, the media, seminars and other available means;

i) encourage the efforts of non - Governmental organizations and institutions working in the field of human rights;

j) such other functions as it may consider necessary for the promotion of human rights.

3.7k views
224. In the 10th Mandala of the Rigveda, which one of the following hymns reflects upon the marriage ceremonies?

(a) Surya Sukta

(b) Purusha Sukta

(c) Dana Stutis

(d) Urna Sutra

B

3.6k views
226. Which one of the following statements about transport mechanism in plants is correct?

a) Diffusion of ions and small molecules across cell membrane requires energy.
b) Active transport of ions and small molecules across cell membrane against concentration gradient requires energy.
c) Cells conserve energy and no energy is required for transport of ions and small molecules against concentration gradient.
d) Cells do not transport ions across their membrane as these are charged molecules.
3.6k views
227.  In mid-latitude regions, the diurnal variation in weather is due to

a) raising air/convection
b) sinking air/conduction
c) advection
d) radiation
3.6k views
228. Springs are common in which of the following areas?

1. Well jointed rocks
2. Arid areas with underlying rocks
3. Karst topography
4. Tilted strata

Select the correct answer using the code given below.

a) 1 and 3 only
b) 1, 3 and 4
c) 2 and 4
d) 3 and 4 only
3.6k views
229. Match List-I with List-II and select the correct answer using the code given below the Lists :

List-I (Limestone region) List-II (Country)

A. Pennines               1. France
B. Yucatan Peninsula 2. Jamaica
C. Cockpit Country    3. Britain
D. Causses                 4. Mexico

Code :
a) A B C D 3 4 2 1
b) A B C D 1 2 4 3
c) A B C D 3 2 4 1
d) A B C D 1 4 2 3
3.6k views
230. Which of the following cities experience the warm temperate Mediterranean climate?

1. Cape Town
2. Los Angeles
3. Adelaide
4. Santiago

Select the correct answer using the code given below.

a) 1 and 2 only
b) 3 and 4 only
c) 1, 2 and 3 only
d) 1, 2, 3 and 4
3.6k views
231. Arabian Sea records higher salinity than the Bay of Bengal because

a) Arabian Sea exhibits high rate of evaporation and low influx of fresh water
b) Arabian Sea has shallow water
c) Arabian Sea has more enclosed land
d) Arabian Sea receives more rainfall
3.6k views
232. Which of the following statements are appropriate to Mangrove Sites?

1. Mangrove plants require appropriate mix of saline water and freshwater.
2. Mangrove plants require mudflats to enable it to grow and develop.
3. Mangrove plants are found in the inter-tidal zones of sheltered coasts.
4. Mangrove vegetation has been reported in all the coastal States including Andaman and Nicobar Islands.

Select the correct answer using the code given below.

a) 1 and 2 only
b) 2, 3 and 4 only
c) 1, 3 and 4 only
d) 1, 2, 3 and 4
3.6k views

mhs11said

226. Which one of the following statements about transport mechanism in plants is correct?

a) Diffusion of ions and small molecules across cell membrane requires energy.
b) Active transport of ions and small molecules across cell membrane against concentration gradient requires energy.
c) Cells conserve energy and no energy is required for transport of ions and small molecules against concentration gradient.
d) Cells do not transport ions across their membrane as these are charged molecules.

B? B and C are opposite statements , so answer should be one of them?

B

3.5k views

mhs11said

230. Which of the following cities experience the warm temperate Mediterranean climate?

1. Cape Town
2. Los Angeles
3. Adelaide
4. Santiago

Select the correct answer using the code given below.

a) 1 and 2 only
b) 3 and 4 only
c) 1, 2 and 3 only
d) 1, 2, 3 and 4

A?

D

3.4k views

mhs11said

231. Arabian Sea records higher salinity than the Bay of Bengal because

a) Arabian Sea exhibits high rate of evaporation and low influx of fresh water
b) Arabian Sea has shallow water
c) Arabian Sea has more enclosed land
d) Arabian Sea receives more rainfall

A

A is correct. 

3.5k views

mhs11said

232. Which of the following statements are appropriate to Mangrove Sites?

1. Mangrove plants require appropriate mix of saline water and freshwater.
2. Mangrove plants require mudflats to enable it to grow and develop.
3. Mangrove plants are found in the inter-tidal zones of sheltered coasts.
4. Mangrove vegetation has been reported in all the coastal States including Andaman and Nicobar Islands.

Select the correct answer using the code given below.

a) 1 and 2 only
b) 2, 3 and 4 only
c) 1, 3 and 4 only
d) 1, 2, 3 and 4

D?

B

3.4k views
229. Match List-I with List-II and select the correct answer using the code given below the Lists :

List-I (Limestone region) List-II (Country)

A. Pennines               1. France
B. Yucatan Peninsula 2. Jamaica
C. Cockpit Country    3. Britain
D. Causses                 4. Mexico

Code :
a) A B C D 3 4 2 1
b) A B C D 1 2 4 3
c) A B C D 3 2 4 1
d) A B C D 1 4 2 3

A

3.4k views
228. Springs are common in which of the following areas?

1. Well jointed rocks
2. Arid areas with underlying rocks
3. Karst topography
4. Tilted strata

Select the correct answer using the code given below.

a) 1 and 3 only
b) 1, 3 and 4
c) 2 and 4
d) 3 and 4 only

B

3.4k views
227.  In mid-latitude regions, the diurnal variation in weather is due to

a) raising air/convection
b) sinking air/conduction
c) advection
d) radiation

C

3.4k views
228. Springs are common in which of the following areas?

1. Well jointed rocks
2. Arid areas with underlying rocks
3. Karst topography
4. Tilted strata

Select the correct answer using the code given below.

a) 1 and 3 only
b) 1, 3 and 4
c) 2 and 4
d) 3 and 4 only

D

Answered all Questions above.

3.6k views
233. Which of the following statements about tropical cyclone are correct?

1. It originates and develops over warm oceanic surface.
2. Presence of Coriolis force is necessary for it.
3. It occurs in middle latitudinal region.
4. It develops “eye” with calm and descending air condition.

Select the correct answer using the code given below.

(a) 1 and 2 only
(b) 1, 2 and 4
(c) 3 and 4 only
(d) 2, 3 and 4
3.5k views
234. Which of the following statements about biodiversity hot spots is/are correct?

1. Biodiversity hot spots are identified by the International Union for Conservation of Nature and Natural Resources (IUCN).
2. Biodiversity hot spots are defined according to their vegetation.
3. In India, Eastern Ghats and Western Himalaya are the biodiversity hot spots.

Select the correct answer using the code given below.

(a) 1 and 2 only
(b) 3 only
(c) 1, 2 and 3
(d) 1 only
3.5k views
235. Which one of the following rivers is west flowing?

a) Godavari
b) Periyar
c) Tungabhadra
d) Cauvery
3.5k views
236. Which of the following statements about town planning in British India in early 19th century is/are correct?

1. The funds for town improvement were also raised through public lotteries.
2. The threats of epidemics gave an impetus to town planning in the early decades of 19th century.

Select the correct answer using the code given below.

a) 1 only
b) 2 only
c) Both 1 and 2
d) Neither 1 nor 2
3.5k views
237. Which one of the following Committees of the Parliament has no Members from the Rajya Sabha?

(a) Public Accounts Committee
(b) Committee on Public Undertakings
(c) Estimates Committee
(d) Departmentally Related Standing Committee (DRSC) on Finance
3.5k views
238. Which of the following statements about Jainism is/are correct?

1. The most important idea in Jainism is that the entire world is animated, even stones, rocks and water have life.
2. Asceticism and penance are required to free oneself from the cycle of Karma

Select the correct answer using the code given below.

(a) I only
(b) 2 only
(c) Both 1 and 2
(d) Neither 1 nor 2
3.5k views
239. Which of the following statements about Bodhisattas is/are correct?

1. They were perceived as deeply compassionate beings who accumulated merit through their Efforts.
2. They made attempt to attain Nibbana.

Select the correct answer using the code given below.

(a) 1 only
(b) 2 only
(c) Both 1 and 2
(d) Neither 1 nor 2
3.5k views
240. Which one of the following statements relating to the power of the President of India to grant pardon is not correct?

(a) The President has the power to grant pardon where punishment or sentence is by a Court Martial.
(b) The President can grant pardon in all cases where the sentence is a sentence of death.
(c) The Court's power of judicial review is very limited in relation to the decision of the President on mercy petition.
(d) The power to grant pardon by the President is the power that the sovereign never exercises against its own judicial mandate.
3.5k views

Agogsaid

@upsc2020 frm wher 2 read abt these lakes?Never hrd!!


You just have to know Bhimtal and Lonar. That's available in several texts. 

One has to learn to eliminate for pre. We can't know everything. :)

Lonar lake was in news but what about Bhimtal lake except the fact that its in Uttarakhand??

Major lakes to we read na. Bhimtal is important to know. 

4.2k views

Microbial fuel cells are considered a source of sustainable energy. Why?1. They use living organisms as catalysts to generate electricity from certain substrates.
2. They use a variety of inorganic materials as substrates.
3. They can be installed in waste water, treatment plants to cleanse water and produce electricity.
Which of the statements given above is/are correct?
(a.) 1 only
(b.) 2 and 3 only
(c.) 1 and 3 only
(d.) 1, 2 and 3


C

4.2k views

Agogsaid

@upsc2020 Hind Mazdoor Sabha or Swarajya Sabha source??


Didn't understand your question. 

4k views
From the point of view of evolution of living
organisms, which one of the following is the correct
sequence of evolution?
(a) Otter – Tortoise – Shark
(b) Shark – Tortoise – Otter
(c) Tortoise – Shark – Otter
(d) Shark – Otter – Trotoise
Can someone explain why its answer  is option b

Don't know. But a simple Fish - amphibian/reptile - mammal sequence can be used. But that's just a general rule. Could be wrong at cases. 

We have solved this question on the previous pages. The general rule how to solve these questions is this image: 


7.6k views

Donnasaid

Reviving this thread with CDS 2021

Great Work!

3.6k views

Donnasaid

Identify the soil on the basis of given characteristics:

1. Rich in Iron, lime, alumina, magnesia.

2. Generally clayey, deep and impermeable.

3. Mainly found in MP, MH, GJ.

Select the correct answer.

a. Laterite soil

b. Red and yellow soil.

c. Black Soil

d. Saline soil

C - Black Soil.

3.6k views

Donnasaid

The situation where the equilibrium level of real GDP falls short of potential GDP is known as

a. Inflationary Gap

b. Recessionary GAp

c. Demand side inflation

d. Supply side inflation

B - Recessionary Gap

3.6k views

Donnasaid

Which sea port gained significance for handling iron ore exports to Japan?

a. Mormugao

b. Kochi

c. Ennore

d. Kandla


None? I thought it was Vizag

3.6k views
3.4k views

When was the monopoly of China trade lost by East India Company?

a. 1813

b. 1833

c. 1838

d. 1860

3.4k views

Which of the following statement is not correct with respect to ryotwari settlement?

a. it was adopted in southern and western India. 

b. it was in principle a direct contract between ryot and state.

c. it means a tax contract valid for usually 30 years

d. it strengthened the zamindars and weakened the peasantry. 

3.4k views

Which one of the following is not a function of money?

a. acts as an internediate in the exchange process

b. acts as a store of value

c. used as the unit of account

d. used for regulating consumption

3.7k views

Which one of the following statements is not correct about British Indian Medical Service?

a. IMS began in 1764

b. It recruited health professionals by means of a competitive exam.

c. the IMS was meant to first look after the troops

d. Indians were never admitted to the IMS.

3.7k views

Which of the following statements are correct about casting vote in Parliament?

1. It is cast by speaker or person sitting as such.

2. It is cast in addition to voting in first instance.

3. It is cast in case of equality of votes.

4. It is always cast to maintain the status quo. 

Select the correct answer.

a. 1, 2 and 3

b. 1 and 3

c. 2 and 4

d. 3 only

3.7k views

Milpa and Ladang are different names for

a. Shifting Cultivation

b. Mixed Farming

c. Truck Farming

d. PLantation Agriculture. 

3.7k views

Correct. 

3.7k views

Mishansaid

When was the monopoly of China trade lost by East India Company?

a. 1813

b. 1833

c. 1838

d. 1860

b

Correct. 

Charter act of 1813 - Ended the trade monopoly of East India Company except it's trade with China and trade in tea

Charter act of 1833 - Ended the Company's monopoly over trade with China as well as it's trade in tea.

3.7k views

Agogsaid

@upsc2020 can the 2 prelims thread b combined plz?


Which two ?

3.6k views

Agogsaid

@upsc2020 1 of previous year ques and anothr just 4 prelims ques.


I think it would be more convenient this way. One only for previous year UPSC questions 

3.5k views

Ankitasaid

Milpa and Ladang are different names for

a. Shifting Cultivation

b. Mixed Farming

c. Truck Farming

d. PLantation Agriculture. 


A

The ‘slash and burn’ agriculture is known as ‘Milpa’ in Mexico and Central America, ‘Conuco’ in Venzuela, ‘Roca’ in Brazil, ‘Masole’ in Central Africa, ‘Ladang’ in Indonesia, ‘Ray’ in Vietnam.

In India, this primitive form of cultivation is called ‘Bewar’ or ‘Dahiya’ in Madhya Pradesh, ‘Podu’ or ‘Penda’ in Andhra Pradesh, ‘Pama Dabi’ or ‘Koman’ or Bringa’ in Odisha, ‘Kumari’ in Western Ghats, ‘Valre’ or ‘Waltre’ in South-eastern Rajasthan, ‘Khil’ in the Himalayan belt, ‘Kuruwa’ in Jharkhand, and ‘Jhumming’ in the North-eastern region.

3.3k views

Which of the following statements are correct about casting vote in Parliament?

1. It is cast by speaker or person sitting as such.

2. It is cast in addition to voting in first instance.

3. It is cast in case of equality of votes.

4. It is always cast to maintain the status quo. 

Select the correct answer.

a. 1, 2 and 3

b. 1 and 3

c. 2 and 4

d. 3 only

b

Correct. 

Speaker never votes in first instance. If he is not acting as a speaker, then only can vote in first instance. 

3.3k views

Which one of the following statements is not correct about British Indian Medical Service?

a. IMS began in 1764

b. It recruited health professionals by means of a competitive exam.

c. the IMS was meant to first look after the troops

d. Indians were never admitted to the IMS.

D

Established in 1764 in British India, Indian Medical Service was a military medical service which also had civilian functions. It was abolished in August 1947. The Mudaliar Committee had in 1961 recommended the creation of a central service.

3.3k views

Which one among the following is a free living animal?

a. Liver fluke

b. Wuchereria

c. Plasmodium

d. Planaria

3.3k views

The Luhri hydro electric project is being constructed on the river?

a. Sutlej

b. Beas

c. Ravi

d. Chenab

3.3k views

A child receives tall plant from father as a birthday gift. Father asked how she could verify whether this tall plant was a progeny of both tall plants or one tall and short plant. She could verify this through

a. Cross Pollination

b. Self Pollination

c. Tissue Culture

d. Negative Propogation

3.3k views

Ellensaid

The Luhri hydro electric project is being constructed on the river?

a. Sutlej

b. Beas

c. Ravi

d. Chenab

A

Located on River Satluj in Shimla and Kullu districts of Himachal Pradesh

Correct.

4.5k views

Ellensaid

Which one among the following is a free living animal?

a. Liver fluke

b. Wuchereria

c. Plasmodium

d. Planaria

What is a free living animal?

An organism that is not directly dependent on another organism for survival. 

Planaria is a nonparasitic or free-living platyhelminth.

4.4k views

Ellensaid

A child receives tall plant from father as a birthday gift. Father asked how she could verify whether this tall plant was a progeny of both tall plants or one tall and short plant. She could verify this through

a. Cross Pollination

b. Self Pollination

c. Tissue Culture

d. Negative Propogation

b?

Correct. 

Pollination is basically a method of sexual reproduction in plant, miosis separates chromosomes and these chromosomes meets again during process of reproduction. Cross pollination or self pollination is method of mixing the chromosomes. Gene of particular trait might be dominant or recessive character. In case of self pollination you get all plant of tall trait, it means plant of progeny of tall plant, in case you get mixed traits(short or long) it is progeny of both tall and short plant.

4.4k views
An antibiotic is not useful against virus whereas a vaccine is. Which one of the following is the most appropriate reason for this?

A. An antibiotic can break RNA only whereas virus has DNA.

B. An antibiotic is a carbohydrate in its chemical nature, whereas a vaccine is a protein, which works well to kill a virus.

C. Only vaccine can break the genetic material of a virus.

D. A virus does not use biochemical pathways which can be blocked by an antibiotic. But a vaccine can boost an immune system to fight the virus.
4.4k views
Which of the following statements with regard to the creation of a new state or alteration of boundaries of states is/are correct?

1. An amendment has to be moved in the Parliament under Article 368 of the Constitution.

2. Ratification by one half of the states is necessary after the amendment is made by the Parliament.

3. The legislation can be passed by a simple majority of both Houses of Parliament.

Select the correct answer:

a. 2 only
b. 1 and 3 only
c. 3 only
d. 2 and 3 only
4.4k views

A

4.4k views
Which of the following lakes is/are situated in Ladakh?

1. Tso Kar

2. Pangong Tso

3. Tsomgo

4. Tso Moriri

6.6k views
An antibiotic is not useful against virus whereas a vaccine is. Which one of the following is the most appropriate reason for this?

A. An antibiotic can break RNA only whereas virus has DNA.

B. An antibiotic is a carbohydrate in its chemical nature, whereas a vaccine is a protein, which works well to kill a virus.

C. Only vaccine can break the genetic material of a virus.

D. A virus does not use biochemical pathways which can be blocked by an antibiotic. But a vaccine can boost an immune system to fight the virus.

( CDS 1 2021)
6.6k views

The percentage by which the money the borrower pays back exceeds the money that was borrowed is called:

a. bank rate

b. nominal interest rate

c. real interest rate

d. terms of credit

6.6k views
Match list 1 to 2 and select the correct answer using the codes given below:

List 1                       List 2

A.Cyclones        1.Western Australia

B.Hurricane        2.South China Sea

C.Typhoons         3.Indian Ocean

D.Willy-Willies      4. Atlantic Ocean 
6.6k views
previous year paper up to which year must be done. do we to that 25 years?

At least for the static subjects like History, Geography, Polity - solve prev 20/25 years.

5.3k views
Write your comment…